Сохранен 535
https://2ch.hk/spc/res/707999.html
24 декабря Архивач восстановлен после серьёзной аварии. К сожалению, значительная часть сохранённых изображений и видео была потеряна. Подробности случившегося. Мы призываем всех неравнодушных помочь нам с восстановлением утраченного контента!

Тред тупых вопросов №161

 Аноним 06/03/22 Вск 04:50:51 #1 №707999 
space-night-tree-wallpaper-preview.jpg
685189909(2).jpg
154466002013752502.jpg
Podrobnaya-klassifikatsiya-meteoritov.jpg
Тред вопросов о жизни, Вселенной и всём таком.

Спрашиваем то, за что в других местах выдают путёвку в биореактор. Здесь анонимные учёные мирового уровня критически рассмотрят любые гениальные идеи и нарисованные в Paint схемы.

Предыдущий тут: >>704732 (OP) (OP)
Аноним 06/03/22 Вск 05:41:09 #2 №708002 
Тут есть люди, которые заканчивали астрономические/физ факультеты?
Аноним 06/03/22 Вск 14:33:47 #3 №708019 
>>707999 (OP)
>4 пик
А как определяют, что ебанный камушек прилетел с Луны или Марса?

И может ли кусок микроастероид в результате ударе об атмосферу войти на стабильную орбиту вокруг земли?
Аноним 06/03/22 Вск 14:47:50 #4 №708020 
>>708002
Да.
Аноним 06/03/22 Вск 16:39:33 #5 №708030 
>>708020
Как учились в школе? Как выбрали себе Вуз?

Как вообще становятся физиками/астрономами?

Кем работаете сейчас?
Аноним 06/03/22 Вск 19:21:56 #6 №708037 
>>708019
>И может ли кусок микроастероид в результате ударе об атмосферу войти на стабильную орбиту вокруг земли?
нет, потому что его перигей будет в атмосфере и на следующем витке он упадёт или сгорит
Аноним 06/03/22 Вск 21:29:39 #7 №708041 
>>708030
>Как учились в школе?
Средне. Но школа была хорошая, приличный физмат.

>Как выбрали себе Вуз?
Просто куда прошёл из топовых по физике в ДС. (А это МФТИ, физфак МГУ, физфак ВШЭ, МИФИ)

>Как вообще становятся физиками/астрономами?
Учатся на физфаке, потом идут работать по специальности. Или уезжают за бугор получать нормальный PhD и остаются там. У меня есть знакомые, которые в США и во Францию так уехали.

>Кем работаете сейчас?
Ну а ты как думаешь? Правильно, айтишником.
Аноним 07/03/22 Пнд 07:35:49 #8 №708052 
>>708041
Не жалеешь, что перекатился из ученого в айтишники?
Если бы не финансовый вопрос, то остался ученым?
Аноним 07/03/22 Пнд 14:34:36 #9 №708067 
Carl-Sagan-Portrait-Scientific-Greats-Sam-Falconer-Illustra[...].jpg
>>708041
Бро, расскажи, как проходит рабочий день астронома, ведь за всей этой романтикой из наблюдений галактик и туманностей стоит рутинная работа из вычислений, которые порой надоедают, плоть до выгорания - я всегда восхищался аутистами, которые готовы посвятить всю свою жизнь подобной работе, но в то же время было безумно интересно, а как проходит их рабочий день.
Аноним 07/03/22 Пнд 21:01:01 #10 №708091 
Может ли уголь образовываться на других планетах?
Аноним 07/03/22 Пнд 21:33:20 #11 №708093 
>>708091
Внезапно да. Но это будет не привычный ископаемый уголь, а скорее рассеянные сажа/графит/алмазы.
Аноним 08/03/22 Втр 11:43:59 #12 №708131 
>>708067
Каждую ночь, надев расшитый звездами плащ и остроконечную шляпу, с астролябией в руках, поднимаешься на башню. Считаешь звезды, прислушиваешься к музыке небесных сфер. А под утро при свете свечи записываешь свои наблюдения в большой фолиант "О наблюдении светил небесных и сопряжениях сфер мировых кои в великой гармонии пребывают"
Аноним 08/03/22 Втр 12:31:42 #13 №708132 
>>708131
Лампово.
Выпью за твое здоровье!
Аноним 08/03/22 Втр 13:29:50 #14 №708134 
image.png
>>708067
Зависит от направления.
Есть преподавательская деятельность, есть прикладная, есть теоретическая.
В прикладной крайне мало присутствия в обсерваториях и еще меньше смотрения в телескоп.
Прикладная по направлениям тоже различается, в дальней радиотелескопы зачастую задействованы, в них вообще не посмотришь.
Есть куча всего, и ничего из того давным давно не представляет то что тут написано >>708131, а жаль.
Аноним 08/03/22 Втр 16:40:00 #15 №708141 
undefined
Навернул картины Курнина и задался вопросом. Может ли планета освещаться (или даже обогреваться) светом от центра галактики, если она будет планетой сиротой или планетой, вращающейся вокруг звезды, несущейся где-то около центра глактики
Аноним 08/03/22 Втр 16:50:45 #16 №708142 
>>708141
>Может ли планета освещаться (или даже обогреваться) светом от центра галактики, если она будет планетой сиротой или планетой, вращающейся вокруг звезды, несущейся где-то около центра глактики
Чтобы прям освещать целую планету, нужно очень много фотонов, а от центра Галактики к планете прилетает мало фотонов. А планета у звезды и будет освещаться светом звезды, а не светом центра Галактики.
Аноним 08/03/22 Втр 16:53:46 #17 №708143 
>>708142
>Чтобы прям освещать целую планету, нужно очень много фотонов, а от центра Галактики к планете прилетает мало фотонов.
А если планета близко к центру галактики?
>А планета у звезды и будет освещаться светом звезды, а не светом центра Галактики.
Планета может быть приливнозахвачена своим карланом и тогда пол-планеты никогда не будет видеть света своей же звезды
Аноним 08/03/22 Втр 20:14:02 #18 №708151 
>>708141
Центр галактики не так чтобы прям очень плотнее нецентра.
Звезды все равно очень далеко друг от друга и тебя.
Так что >>708143 даже если ты в центре и не приливно связан со звездой, то от тебя до звезд световые года. Т.к. освещенность падает квадратически, то одна такого освещения и тепла как у родной звезды ты не добьешься.
Может быть можно о чем-то говорить если твою планету квазар будет освещать, это уже другое дело.
Аноним 08/03/22 Втр 23:56:12 #19 №708165 
Есть ли крупные небесные тела, больше чем условная луна, которые свободно летают и не принадлежат к планетарной системе?
Могут ли столкнутся две планетарные системы?
Аноним 09/03/22 Срд 05:55:48 #20 №708174 
>>708165
https://ru.wikipedia.org/wiki/Планета-сирота
Аноним 09/03/22 Срд 12:24:28 #21 №708184 
>>708165
Да, и похоже их на порядок больше чем звезд. Это обусловлено тем, что звезды образуются плотными группами, а потом за счет взаимного гравитационного возмущения разбегаются, в таких условиях тела на границах диска вокруг звезды мелкие тела выкидываются из системы.
Могут даже звезды столкнуться с друг другом, но такие события чрезвычайно редкие. С планетами-сиротами вероятность еще ниже.
Аноним 09/03/22 Срд 13:17:26 #22 №708189 
>>708143
>А если планета близко к центру галактики?
Центр - понятие растяжимое. Если прям около черной дыры крутится, то еще может освещаться аккреционным диском, а если "где-то примерно в центре", то точно нет.
>Планета может быть приливнозахвачена своим карланом и тогда пол-планеты никогда не будет видеть света своей же звезды
Да, но центр Галактики ей с этим никак не поможет.
>>708165
>Есть ли крупные небесные тела, больше чем условная луна, которые свободно летают и не принадлежат к планетарной системе?
Да, есть блуждающие планеты.
>Могут ли столкнутся две планетарные системы?
Нет.
Аноним 09/03/22 Срд 17:08:48 #23 №708205 
Если в космосе есть всякие алмазные или золотые планеты то может ли быть планета из мяса?
Аноним 09/03/22 Срд 18:53:42 #24 №708210 
16422431779680.png
>>708205
>планета из мяса?
Ну, кстати, вполне возможно. Первые организмы на Земле были неподвижными, плоскими и лежали на дне. Гипотетически на другой планете такие штуки могли разрастись по всей поверхности и остановить эволюцию на миллиарды лет. Таким образом появится мясная планета. Но только поверхность будет мясной, естественно.
Аноним 09/03/22 Срд 19:27:15 #25 №708213 
>>708205
> Если в космосе есть всякие алмазные или золотые планеты то может ли быть планета из мяса?
Мясо это двигатель, работающий на АТФ. Что и нахуя ему двигать? Ни в фотосинтез, ни во всё остальное оно не умеет.
Может быть планета, покрытая многокилометровым слоем растений и какой-нибудь хуйни типа грибов.

>>708165
>Могут ли столкнутся две планетарные системы?
Могут, причём через 1.4млн лет у нас есть шансы на такое столкновение нарваться. Гугли Глизе 710.

Поскольку планетарная система, подобно атому, в основном состоит из сплошного нихуя - при столкновении они проходят сквозь друг друга. Но вот эффекты зависят от того, насколько близко сблизятся центры. Скажем, при сближении на 1000а.е. начинается коментный беспредел, на 100а.е. уже может чувствительно пошатать планетам орбиты, а на 10а.е. планеты сорвёт нахуй, перекроив обе системы самым непредсказуемым образом.

Аноним 09/03/22 Срд 19:37:27 #26 №708214 
565401original.jpg
>>708189
>Центр - понятие растяжимое.
Чтобы вот как-то так как на этой картинке
>>708205
Полностью на все 100% объёма твёрдого тела планеты? Нет. Можно в теории встретить планету, 100% поверхности которой покрыто колонией грибов или каких-нибудь зооидов
Аноним 09/03/22 Срд 22:32:10 #27 №708219 
image.png
>>708214
>Чтобы вот как-то так как на этой картинке
Это настолько далеко от центра что ты уже давно не в галактике.
Аноним 10/03/22 Чтв 03:19:59 #28 №708227 
Когда близко к земле будет подлетать астероиды, потенциально угрожающие столкновением? Как называются и в каком году приблизительно?
Аноним 10/03/22 Чтв 03:55:47 #29 №708228 
>>708227
https://en.wikipedia.org/wiki/List_of_asteroid_close_approaches_to_Earth
Самый близкий будет гипофиз в 29 году, 38 мегаметров от земли (считай геостационарка).
Аноним 10/03/22 Чтв 07:32:59 #30 №708229 
image.png
Какие есть игры про мирный космос? Хочется лампово развиваться, исследовать, колонизировать. Войну не хочу.
Огурцов до дыр наиграл.
Аноним 10/03/22 Чтв 10:02:11 #31 №708233 
Dyson Sphere Program Trailer.mp4
>>708229
Аноним 10/03/22 Чтв 13:01:10 #32 №708241 
Мне приснился сон и возник вопрос. Вокруг звезд есть же зоны обитаемости или зоны жидкой воды. Что это значит? Имеется в виду только вода на поверхности какой-либо планеты? Или допустим просто висящая в космосе вода может быть жидкой (не замерзать и не испаряться) на определенном расстоянии от звезды? Теоретически где-нибудь в космосе может быть вокруг звезды кольцо из жидкой воды без какой-либо планеты и твердой поверхности?
Аноним 10/03/22 Чтв 14:06:28 #33 №708242 
>>708241
В вакууме вода существует либо в виде льда либо в виде пара.
Аноним 10/03/22 Чтв 14:32:34 #34 №708243 
>>708229
Per Aspera
Oxygen Not Included
Surviving Mars
Space Engineers
Planetbase
Аноним 10/03/22 Чтв 16:27:53 #35 №708249 
>>708214
На пикче вообще хуйня бессмысленная изображена. Типа с поверхности какой-то планеты невооруженным взглядом видны спиральные галактики с отчетливо различимой спиральной структурой. Бред.
>>708241
>Что это значит?
Значит, что там не слишком холодно и не слишком жарко - температура достаточная для того, чтобы на поверхности могла существовать вода в жидком состоянии.
Аноним 10/03/22 Чтв 17:28:42 #36 №708253 
В центре нашей галактики есть активные Чд?
Аноним 10/03/22 Чтв 17:54:09 #37 №708255 
>>708249
>Типа с поверхности какой-то планеты невооруженным взглядом видны спиральные галактики с отчетливо различимой спиральной структурой. Бред.
Не бред. Запросто можно.
Аноним 10/03/22 Чтв 18:03:42 #38 №708256 
>>708253
Не считая СЧД полно гамма источников.
Аноним 10/03/22 Чтв 18:05:02 #39 №708257 
>>708256
Так есть Чд которые прям щас жрут что либо?
Аноним 10/03/22 Чтв 18:23:55 #40 №708259 
Казань. Прямо щас 18-23 мск, примерон на юго-западе,ооккло 25-30 градусов над горизонтом, ярко мерцака аки диско-шар звезда. Что это за звезда и почему другие так не мерцают?
Аноним 10/03/22 Чтв 18:30:46 #41 №708260 
image.png
>>708259
Разве что сириус там где ты указал.
https://stellarium-web.org можешь сам поискать.
Аноним 10/03/22 Чтв 18:38:41 #42 №708263 
>>708259
Турбулентность воздуха
Аноним 10/03/22 Чтв 22:22:28 #43 №708273 
>>708257
Собственно все гамма источники, это аккреционые диски ЧД или охуевшие нейтронные звезды. Первых больше.
Аноним 11/03/22 Птн 01:08:49 #44 №708284 
Как понять астрономию? Не особо интересует "математика" астрономии, но хотя бы понимать значения слов этой науки хочется.
Аноним 11/03/22 Птн 02:02:15 #45 №708287 
>>708284
>понимать значения слов
Ты не пробовал интересоваться теми словами, которые не понимаешь? Ну там википедию читать, например...
Аноним 11/03/22 Птн 03:10:46 #46 №708288 
>>708287
Википедию на тему астрономии пишут слишком сложно. У меня в школе не было астрономии, поэтому трудно воспринимать текст.
Аноним 11/03/22 Птн 03:30:08 #47 №708289 
>>708288
Ну тут спрашивай непонятные слова, это же тред тупых вопросов.
Поясним как для двачера.
Аноним 11/03/22 Птн 15:10:04 #48 №708318 
>>708253
Есть одна сверхмассивная черная дыра и она не очень активна.
>>708255
Ага, а еще можно розовых единорогов наблюдать.
Аноним 11/03/22 Птн 17:51:09 #49 №708331 
>>708318
У розовой еще и крылья есть, так что она не единорог, а аликорн.
А галактики прррекрасно видно невооруженным взглядом.
То что ты их ни разу не видел лишь говорит о тебе в печальном ключе.
Аноним 11/03/22 Птн 18:31:20 #50 №708334 
>>708318
Езжай в южное полушарие в горы, вдали от городов посмотри на млечный путь. А потом представь, что ты на эту ебанину не с торца смотришь.
Аноним 11/03/22 Птн 20:31:47 #51 №708341 
>>708318
чёрная дыра - она как одинокий пидарас, если рядом никого нет то не активен и печален.
Аноним 12/03/22 Суб 08:52:50 #52 №708371 
Существуют ли Белые Дыры?
Аноним 12/03/22 Суб 09:47:51 #53 №708372 
>>708371
Крайне маловероятно.
Аноним 12/03/22 Суб 09:48:29 #54 №708373 
>>708372
А что мешает им существовать?
Аноним 12/03/22 Суб 10:01:27 #55 №708374 
image.png
>>708373
В привычном понимании нормальной метрики пространства-времени такие объекты просто появиться во вселенной не могут, гравитация сонаправлена со временем, потому нет механизмов для их появления.
Гипотетически предполагалось наличие первичных белых дыр образовавшихся со вселенной, но нет таких наблюдаемых объектов.
Можно, на самом деле, просто взглянуть с другой стороны, тогда любая ЧД является белой.
Если английский знаешь, могу пару видео подкинуть, поясняют про вопрос:
PBS SpaceTime: https://youtu.be/S4aqGI1mSqo
Или попроще: https://youtu.be/p3P4iKb24Ng
Аноним 12/03/22 Суб 10:56:16 #56 №708378 
>>708374
А сингулярность можно достичь с помощью ИИ?
Аноним 12/03/22 Суб 11:34:39 #57 №708381 
image.png
>>708378
Технологическую или математическую? Или какую еще?
Технологическую - примерно так и предполагается, что ИИ может быть шагом к таковой.
Математическая давно выведена, это ещё в школе вроде давали.
y=1/x имеет сингулярность в нуле, например.
Аноним 12/03/22 Суб 15:16:30 #58 №708401 
В чём проблематика создания орбитального стратегического бомбардировщика?
Аноним 12/03/22 Суб 15:32:03 #59 №708402 
>>708331
>То что ты их ни разу не видел лишь говорит о тебе в печальном ключе.
Тебе с боярой надо завязывать, долбоебина.
>>708334
>Езжай в южное полушарие в горы, вдали от городов посмотри на млечный путь. А потом представь, что ты на эту ебанину не с торца смотришь.
Я говорил про отдельные галактики как на пикче, а не про Галактику, которую мы видим изнутри и спиральную структуру которой, кстати, невооруженным глазом увидеть невозможно как раз по этой причине.
>>708371
Нет.
Аноним 12/03/22 Суб 15:52:11 #60 №708403 
>>708402
>Я говорил про отдельные галактики как на пикче, а не про Галактику, которую мы видим изнутри и спиральную структуру которой, кстати, невооруженным глазом увидеть невозможно как раз по этой причине.
Блядь я тебе написал же. Представь, что Солнце находилось бы не в плоскости диска, а на периферии гало галактики и над плоскостью. Или вообще снаружи галактики, но неподалёку от неё.
Аноним 12/03/22 Суб 15:57:47 #61 №708404 
>>708401
В том, что в таком бомбардировщике нет смысла. Его орбита всем известна, его легко сбить, легко отслеживать, при этом он нарушает договор о космосе.
Аноним 12/03/22 Суб 16:59:23 #62 №708405 
>>708404
>Его орбита всем известна
Точно известна? Почему бы не проводить тайные пуски?
Аноним 13/03/22 Вск 05:25:01 #63 №708424 
>>708405
Запуск ракеты не скроешь, их отслеживали еще с 80х годов.
Аноним 13/03/22 Вск 06:29:41 #64 №708426 
Могут ли находиться 2 черные дыры очень близко? Что будет происходить при из сближении? Они сольются в одну или аннигилируют себя и все вокруг? Как бы это выглядело со стороны?
Аноним 13/03/22 Вск 09:05:15 #65 №708429 
>>708241
>Теоретически где-нибудь в космосе может быть вокруг звезды кольцо из жидкой воды без какой-либо планеты и твердой поверхности?
Теоретически, очень теоретически, и с малой даже с сайфай точки зрения вероятностью, может быть в какой-нибудь космоперди - в войде или за пределами галактического рукава - то есть вне, но рядом с основной массой звёзд, может получиться "планета" из воды. Не как в фильме Водный мир и не как в Старбаунде рыбояпонцев, а полностью планетоид на 100% из воды. При условии, что вся эта куча воды наберёт достаточную массу, можно будет получить планету с атмосферой из водяного пара, кору-мантию из жидкой воды и ядро из сверхкритической твёрдой воды и какого-нибудь льда-IX.
Может быть, при таком состоянии планета даже получит что-то типа магнитного поля, но это уже вилами по воде писано.
Аноним 13/03/22 Вск 09:07:59 #66 №708430 
>>708426
>Что будет происходить при из сближении? Они сольются в одну
Да, сольются в одну. И попердолят простарнство-время в ближайшем окружности от себя. Именно за счёт коллапса двух ЧД были пару лет назад зафиксированы гравитационные волны
Аноним 13/03/22 Вск 09:23:02 #67 №708431 
undefined
Есть два вопроса.
1) Внимательно рассмотри кристалл на прилагающейся фотографии. Можете ли вы представить себе, как этот кристалл плачет и набирает номер какой-то невнятной жеоде, чтобы рассказать ей о своих чувствах Я тут задумался, а может ли такой кристалл быть на поверхности суши какой-нибудь планеты? Если да, то как он может там появиться? Размером с гору я думаю таких красавцев не найти, максимум - с какой-нибудь средневековый замок, а минимальные размеры и на Земле можно найти с палец размером.
2) Вот есть у нас, значит-с, Венера. На сегодняшний день нам известно, что там оче плотная атмосфера и внизу, у поверхности, она становится сверхкритической и уже переходит в жидкость. Но то углекислый газ и серный побочки - то есть своя, тяжёлая атмосфера. А у нас на Земле атмосфера-то помягче будет и полегче. Но меня интересует, а что будет с земной атмосферой, если мы просто возмём и Ctrl+C Ctrl+V земную атмосферу и будет так делать до появления у поверхности сверхкритической жидкости земного воздуха. Солнце ещё будет видно с земли? Насколько высоко поднимется уровне атмосферы и поглотит ли она орбитальную станцую с её жалкими 100км высоты? Останется ли воздух прозрачным? Появятся ли явления, аналогичные облакам и клубам дыма внутри массы воздуха Земли, , близки к поверхности? Что будет с дождями, ветрами, ураганами, смерчами, молниями, снегопадами и т.д.? Что будет с растениями и животными? Как на такое жосткое давление сверху отреагирует обычная стандартная поверхностная земная вода?
Аноним 13/03/22 Вск 09:45:17 #68 №708434 
image.png
>>708402
Анончик, сгоняй куда-то хотя бы с третьим уровнем светового загрязнения.
Ты явно даешь понять что неба ты не видел, и это печально.
Оно стоит того, ты охуеешь, я гарантирую. Попробуй, если есть возможность.
Аноним 13/03/22 Вск 14:02:54 #69 №708443 
>>708430
Ну а как оно выглядит, хотя бы примерно? Как взрыв там или где?
Аноним 13/03/22 Вск 14:34:02 #70 №708446 
Почему идея многоразовых ракет не была реализована лет на 20-30 раньше?
Аноним 13/03/22 Вск 15:29:14 #71 №708452 
>>708334
Зачем в южное полушарие, если в европейской части РФ то достаточно на север Архангельской области в пердь заехать, или в Калмыкию какую-нибудь, или есть (пока ещё) очень тёмный закуток около Великих Лук. Лучше где-нибудь ближе к сентябрю, чтобы ночь подлиннее.
Аноним 13/03/22 Вск 15:36:25 #72 №708453 
>>708446
Реализована через жопу - шаттл.
До восьмидесятых боролись за каждый грамм эффективности, после них считали что это всё равно экономически не оправдано. В целом так и было. Это лишь Муск умудрился сделать это эффективным, в условиях конкретной географии и промки. Если бы роскосый не нюхал клей, то может мог бы запилить бы возврат бустеров на крыльях и разгонников на надувных замедлителях, но даже так было не очевидно. В девяностых бил аэроспейс активно работали над биг дамб бустером вместо реюза, может быть это и прокатило бы для не сверхтяжелых ракет, не брось эндрю бил идею при первом запахе проблем.
Аноним 13/03/22 Вск 15:38:34 #73 №708455 
>>708452
>Архангельской
Костромской блять. Совсем обдвачевался.
Аноним 13/03/22 Вск 16:16:48 #74 №708457 
>>708443
>Ну а как оно выглядит, хотя бы примерно? Как взрыв там или где?
https://www.youtube.com/watch?v=vZslsjFaR8Q
Аноним 13/03/22 Вск 16:41:12 #75 №708460 
Когда будут реализованы орбитальные заводы?
Аноним 13/03/22 Вск 17:21:11 #76 №708466 
>>708403
>Представь, что Солнце находилось бы не в плоскости диска, а на периферии гало галактики и над плоскостью. Или вообще снаружи галактики, но неподалёку от неё.
А я тебе написал, что не имею в виду нашу Галактику, с какого бы бока мы ее ни наблюдали. Даже в описанном тобой случае мы бы видели только огромный диск Галактики и ничего более. Других галактик с такой же четкостью мы бы не наблюдали.
>>708426
>Могут ли находиться 2 черные дыры очень близко?
Регулярно регистрируют гравитационные волны от слияния таких близких чд.
>Что будет происходить при из сближении?
Испускание гравитационных волн, и чем ближе они будут, тем сильнее будет гравволновой сигнал.
>Они сольются в одну
Это.
>Как бы это выглядело со стороны?
Если вокруг них нихуя нет, никаких аккреционных дисков, то никак. Ты бы ничего не увидел, они же черные блядь.
>>708434
Еблан, даже на самом чистом небе из галактик (не считая нашей) ты можешь разглядеть лишь Туманность Андромеды и Магеллановы Облака, да и то лишь в качестве очень мутных призрачных пятен на небосводе. Никакой отчетливой спиральной структуры у Туманности Андромеды, как на пикче, ты невооруженным взглядом не различишь. И то, к слову, чтобы разглядеть эти мутные пятна на чистом небе, тебе нужно обладать охуенным 100%-м зрением, что в современном мире дано совсем не каждому.
Аноним 13/03/22 Вск 17:51:32 #77 №708467 
>>708460
Тогда когда запилят экосистему автономных систем без мясных мешков. Короче когда запилят нормальный ИИ и технологических(легко и быстро клепать) роботов.
Аноним 13/03/22 Вск 18:44:42 #78 №708469 
Антон, напомни, пожалуйста, адрес сайта с онлайн-симуляцией движения небесных тел. Это именно онлайн-программа, никакая не селестия, не спейс ориджин, ничего такого. Помню, там совсем простой интерфейс был, звёзды и планеты - простые кружочки. Можно задавать начальные параметры и смотреть как развивается система. Никак не могу найти, может ты помнишь.
Аноним 13/03/22 Вск 22:01:32 #79 №708482 
>>708466
Зачем порвался? Обидно что люди видят галактики а ты ни разу не видел?
Еще раз - съебандос за город оформи, увидишь. Пиздеть потом перестанешь. Хватит позориться.
Аноним 14/03/22 Пнд 04:03:05 #80 №708500 
>>708431
>она становится сверхкритической и уже переходит в жидкость.
Мозг твой переходит в жидкость. На венере вполне себе твёрдая поверхность. 450 градусов цельсия нихуя не достаточно чтобы сделать поверхность жидкой.
Аноним 14/03/22 Пнд 16:16:10 #81 №708546 
>>708482
Я бывал за городом в деревне 100 раз и ни разу ничего подобного не видел, хуйло тупорылое. Ссу тебе на ебало.
Аноним 14/03/22 Пнд 17:30:37 #82 №708559 
Screenshot 2022-03-14 at 17-26-50 Stellarium Web Online Sta[...].png
Screenshot 2022-03-14 at 17-29-45 Stellarium Web Online Sta[...].png
>>708452
В южное полушарие затем, что центр галактики находится в стрельце, а это зимнее созвездие, то есть его видно, когда в северном полушарии лето, солнце поднимается высоко, а противоположная сторона эклиптики ночью низко. В южном полушарии в это время "зима", то есть солнце низко, а стрелец высоко, да и ночь длиньше. Пикрил 1 - стрелец в августе из Москвы, пикрил 2 - та же дата, но из чилийского Сантьяго, взгляд практически в зенит.
Аноним 15/03/22 Втр 01:21:10 #83 №708605 
>>708546
Хуево тебе без зрения, да, бедолага?
Аноним 15/03/22 Втр 07:29:20 #84 №708614 
>>708500
>Мозг твой переходит в жидкость. На венере вполне себе твёрдая поверхность. 450 градусов цельсия нихуя не достаточно чтобы сделать поверхность жидкой.
Ну ты долбоёб пиздоглахый. Речь не по поверхность, расплавленную от жара, а про сжижения углекислого газа у поверхности
Аноним 15/03/22 Втр 13:17:16 #85 №708632 
6939834.jpeg
Пачаны, разве само существование вселенной не противоречит второму началу термодинамики? Откуда появилась вся эта энергия на звезды квазеры и прочее, если она может только убывать?
Аноним 15/03/22 Втр 14:37:53 #86 №708637 
>>708632
Энергия никуда не убывает, во вселенной её сейчас должно быть столько же, сколько во время большого взрыва.
Аноним 15/03/22 Втр 16:05:15 #87 №708643 
>>708637
Он видимо имел в виду энтропию.

>>708632
> разве само существование вселенной не противоречит второму началу термодинамики?
Формально - нет, просто мы не знаем, откуда взялось состояние низкой энтропии в начале существования вселенной. Может быть случайно так сложилось.
Аноним 15/03/22 Втр 17:56:49 #88 №708647 
>>708605
Дегенерат, даже со стопроцентным зрением не будет видно никак того, что на той пикче. Туманность Андромеды видна лишь как очень мутное пятно.
>>708632
>Откуда появилась вся эта энергия на звезды квазеры и прочее
Из вещества.
Аноним 16/03/22 Срд 03:10:41 #89 №708683 
>>708614
>а про сжижения углекислого газа у поверхности
Ясно, поехавший.
Аноним 16/03/22 Срд 10:48:27 #90 №708698 
>>708318
Почему галактики имеют разное строение?
Аноним 16/03/22 Срд 11:39:36 #91 №708704 
Я всего лишь тупой школобубель и своим скудным умишко пытаюсь тут понять физику. Скажите у меня одного возникла мысль что квантовая физика это наебка планетарных масштабов?
Когда открывали деление ядра физики всего мира взаимодействовали открыто и могли проверять результаты в лабораторях.
Но после создания ядерного оружия все как будто держат свои настоящие исследования в тайне и делятся друг с другом дезинформацией, которую называют квантовой физикой, которую не проверишь в лаборатории. И радостно поддакивают друг другу, когда вешают друг другу лапшу на уши.
Может быть такое?
Аноним 16/03/22 Срд 12:03:27 #92 №708711 
>>708704
>которую называют квантовой физикой, которую не проверишь в лаборатории
Вот только она отлично проверяется в лаборатории.
Аноним 16/03/22 Срд 12:04:06 #93 №708712 
>>708698
А пуркуа бы и не па?
Аноним 16/03/22 Срд 12:06:30 #94 №708715 
>>708712
Ту парле франсе?
Аноним 16/03/22 Срд 12:11:36 #95 №708719 
>>708715
Мэхд! Же не ма па сис жур!
Аноним 16/03/22 Срд 12:15:34 #96 №708720 
>>708719
Киса Воробьянинов?
Аноним 16/03/22 Срд 13:54:14 #97 №708728 
>>707999 (OP)
сиськи в космосе торат?
Аноним 16/03/22 Срд 16:20:33 #98 №708740 
>>708698
Потому что по-разному формировались.
>>708704
>Скажите у меня одного возникла мысль что квантовая физика это наебка планетарных масштабов?
Да, потому что ты невежественный долбоеб, который нихуя не знает. Квантовая механика - самая проверенная и точная теория в физике.
Аноним 17/03/22 Чтв 01:29:52 #99 №708770 
>>708704
>Но после создания ядерного оружия все как будто держат свои настоящие исследования в тайне
Любителям теорий заговоров в другой тред.
Все исследования в открытом доступе. Тот же проект ЦЕРН, в котором, лол теперь нашей раши нет и не будет.
Аноним 17/03/22 Чтв 01:53:17 #100 №708771 
>>707999 (OP)
Как найти планету Юггот? У него альбедо меньше, чем у Меркурия, находится он где-то за Нептуном, плюс вращается он перпендикулярно эклиптике.
Аноним 17/03/22 Чтв 07:04:05 #101 №708782 
Я понимаю, откуда луны у гигантов и массивных планет. Я предполагаю, что и у ЧД или нейтронных звёзд могут быть спутники - законом не запрещено, хули. А вот откуда спутники у астероидов милипиздрических (скажем, сам астероид 100м в диаметре, а его спутник - 10м) и почему они летят порой даже без вращения относительно друг друга. Как так? Может ещё труп комсмонавта сделать луной спутника астероида, чтобы он под Сабатон наматывал круги вокруг 10м камушка?
Аноним 17/03/22 Чтв 07:07:01 #102 №708783 
>>708647
Жалко тебя. Ты не только слепой, но еще и тупой.
Аноним 17/03/22 Чтв 08:48:01 #103 №708787 
>>708771
>вращается он перпендикулярно эклиптике
Всё просто, ищешь звёзды, лежащие перпендикулярно эклиптике, наблюдаешь за ними, ждёшь транзит.
Аноним 17/03/22 Чтв 10:32:34 #104 №708792 
>>708782
> Может ещё труп комсмонавта сделать луной спутника астероида, чтобы он под Сабатон наматывал круги вокруг 10м камушка?
Можно.
Если ты достаточно далеко от массивных тел, то у тебя даже атомы могут гравитацию друг друга сильнее ощущать.
У пояса астероидов это все изменчиво и ненадолго. А вот в межгалактическом пространстве хоть Маленького Принца в натуре отыгрывай.
Аноним 17/03/22 Чтв 10:32:45 #105 №708793 
>>708782
Все дело в законе сохранения момента импульса.
Когда у тебя большая куча вещества под действием гравитации превращается в один объект, то куче для превращения надо сбросить лишний момент. Сбрасывать лишний момент довольно сложная задача, это часто энергетики невыгодно. Но есть ход конем - сконденсироваться не в один кусок, а два или более. Хоть момент импульса не броситься, но зато тела смогут в самогравитацию.

Кстати, это причина по которой одиночных звезд меньшинство в галактике.
Аноним 17/03/22 Чтв 15:49:27 #106 №708808 
Поясните, правильно ли я понимаю, что есть определенный предел стартовой массы ракеты с химическими двигателями, выше которой уже практически не получится увеличивать массу полезной нагрузки?
Аноним 17/03/22 Чтв 16:09:52 #107 №708809 
>>708783
>Жалко тебя.
Мамашу свою лучше пожалей, ей вчера очень тяжело было с моим хуем во рту.
Аноним 17/03/22 Чтв 16:25:55 #108 №708810 
>>708809
>пожар пукнутогт петуха
Какая прелесть.
Аноним 17/03/22 Чтв 17:16:59 #109 №708813 
>>708793
Получается, странная форма кометы Чурюмова-Герасименко такая, потому что когда это были большая комета и её спутник, которые просто однажды столкнулись?

А если подумать, если всё так, где луны Марса (я имею в виду нормальные пропорциональные луны, а не две этих кометы-пародии Фобос и Деймос), Венеры, Меркурия? Есть ли луны у Цереры и Весты и прочих поясо-астероидных карланов?
Аноним 17/03/22 Чтв 17:50:26 #110 №708817 
>>708813
Кометы это отдельный разговор.
Они начали формироваться, когда планетезимали начали активно расчищать протопланетный диск, активно конденсируя на себя вещество. Не правильно форма обусловлена хаотическим ростом/испарения льда.

Марсу и прочим Церерам обзавестись крупными Лунами помешал "прыжок" Юпитера, который выбросил много вещества из Системы.

У Меркурия не может быть спутника, поскольку слишком сильный прилив от Солнца. Впрочем Меркурий пережил мощное столкновение и влетел ближе к Солнцу, потеряв много массы.

С Венерой непонятки. Похоже она не переживала крупных столкновений и не мигрировала.
Аноним 17/03/22 Чтв 19:29:26 #111 №708822 
Вопрос такой, кто какими штативами пользуется для астрофотографии? Есть ли годные варианты на алике (ну или в специализированном магазине) в пределах 10к рублей? Хочу купить какой-нибудь штатив, а астротрекер сам сделаю из говна и палок. Из оборудования пока только зеркалка(
Аноним 17/03/22 Чтв 20:14:54 #112 №708829 
Я хочу изучать астрофизику, но не уверен, что хочу быть именно астроном, а не чем-то иным связанным с космосом.
Что можете посоветовать?
Аноним 17/03/22 Чтв 20:21:05 #113 №708831 
>>708829
Иди на теоретического физика и открой уравнение состояния для вещества в нейтронных звездах.
Аноним 17/03/22 Чтв 20:22:57 #114 №708832 
>>708829
Иди на отделение физики физфака МГУ, потом на кафедру физики космоса, дальше поймёшь, что именно тебе интересно.
Аноним 17/03/22 Чтв 20:25:24 #115 №708834 
>>708831
>>708832
Как я понимаю, то в инженера/программиста потом легко переключиться с таким багажом знаний.

Так-то интересно узнать "а что внутри ЧД и есть ли БД".
Аноним 17/03/22 Чтв 20:33:31 #116 №708837 
>>708834
Переключиться можно. Легко будет, если будешь параллельно саморазвиваться в этом направлении, можешь курсы какие-нибудь пройти.

>Так-то интересно узнать "а что внутри ЧД и есть ли БД".
Ну тогда тебе на теорфиз или космологию. Но вообще, что тебе интересно и в каком именно виде, ещё успеет поменяться десять раз, пока ты учишься и общаешься с разными людьми. Главное общаться, со старшекурсниками на разных кафедрах, с научруком, аспирантами, людьми из других вузов и стран. Читать статьи, книжки и так далее.
Аноним 17/03/22 Чтв 21:09:30 #117 №708839 
>>708837
Спасибо. Добра!
Аноним 18/03/22 Птн 08:13:00 #118 №708862 
>>708817
>Похоже она не переживала крупных столкновений
А как ты узнаешь? Там поверхность спеклась вся давно.
Аноним 18/03/22 Птн 08:51:02 #119 №708863 
>>708466
А решили вопрос, что при слиянии двух чд в одну часть пространства которая была которая на момент слияния была в чд перестала там быть?
Аноним 18/03/22 Птн 11:23:43 #120 №708875 
>>708810
То есть твою мамашку выебали в рот, а тебе нравится?
>>708863
Чего блядь?
Аноним 18/03/22 Птн 11:23:44 #121 №708876 
>>708862
Столкновения с крупными телами влияют на орбитальные характеристики и собственное вращение.
У Венеры орбита с очень низким эксцентриситетом. В добавок она очень медленно вращается вокруг своей оси.
Пока что нет данных о сейсмологии и внутреннем строение, но по косвенным данным можно судить, что ее формирование было под постоянном влиянием прилива от Солнца.
Аноним 18/03/22 Птн 11:30:33 #122 №708877 
>>708875
Школьник, попустись нахуй, пока живой
Аноним 18/03/22 Птн 15:20:49 #123 №708895 
изображение.png
>>707999 (OP)
Я нашёл интересную догадку, чую, близко к правде
Аноним 18/03/22 Птн 15:23:10 #124 №708896 
>>708632
Создание игровой вселенной может запросто противоречить законам игровой вселенной, но с позиции создателей игровой вселенной никакого нарушения нету.
Аноним 18/03/22 Птн 22:44:30 #125 №708918 
Наступит ли когда-нибудь время, когда человечество будет строить космические объекты размером хотя бы с четверть Луны? Или это всё маняфантазии?
Аноним 18/03/22 Птн 23:08:21 #126 №708922 
>>708918
Скоро настанет. ИИ запилим и запануем.
Аноним 19/03/22 Суб 01:05:31 #127 №708941 
Вопрос: почему нет треда про поиск экзопланет с дивана?
Аноним 19/03/22 Суб 03:30:22 #128 №708944 
дефицит 4.jpg
Q2p3YfAJ2Ws.jpg
>>708829
>Что можете посоветовать?
В сегодняшних реалиях учиться плести лапти, копать огород и разбираться в сортах картопли.
Аноним 19/03/22 Суб 03:31:53 #129 №708945 
>>708895
хуета хует
самая тупая гипотеза из множества
Аноним 19/03/22 Суб 08:09:14 #130 №708947 
>>708895
Говно гипотеза, на самом деле. Ей для работы необходима "вселенная межзвездных ваншотов", когда построить Звезду Смерти что бы шмалять по планетам можно, а вот кроме Звезды Смерти ничего нельзя. Потому что как только ты не можешь быть уверен, что первым выстрелом прибъешь любого и никто не заметит, палить в каждого встречного становится не лучшей идеей.
Аноним 19/03/22 Суб 09:22:49 #131 №708950 
>>708829
>Что можете посоветовать?
1. Уезжай из страны.
Аноним 19/03/22 Суб 10:24:57 #132 №708957 
Как спутники или другие космические аппараты избавляются от лишнего положительного заряда, вызванный тем, что солнечное излучение вызывает фотоэффект и выбивает электроны?
Аноним 19/03/22 Суб 11:16:40 #133 №708960 
>>708957
>Как спутники или другие космические аппараты избавляются от лишнего положительного заряда, вызванный тем, что солнечное излучение вызывает фотоэффект и выбивает электроны?
Никак
Аноним 19/03/22 Суб 12:51:03 #134 №708967 
>>708877
Свинка, успокойся.
Аноним 19/03/22 Суб 13:08:16 #135 №708971 
Могут ли американцы отрубить Россию от GPS? Насколько я понимаю, в первую очередь они могут отключить наземную инфраструктуру для корректировки и запоганить точность. А могут ли они поднасрать на уровне космоса и в принципе ограничить передачу сигнала над территорией РФ?
Аноним 19/03/22 Суб 14:59:47 #136 №708981 
>>708957
Разматывают длинный шнурок и сбрасывают статику в атмосферу.
Ой, нет, это для свода с орбиты.
Никак.
Аноним 19/03/22 Суб 18:28:05 #137 №708995 
>>708967
Я тебя успокою лучше, давай геолокацию свою.
Аноним 19/03/22 Суб 18:59:56 #138 №708996 
image.png
Я читаю-читаю, и нихуя не вдупляю, слишком тупенький, видимо.
Может мне кто-нибудь разъяснить про хиральное нарушение симметрии? Я знаю, что на дваче с помощью говна и мочи могут самые тяжелые концепции донести, получится ли тут?
Аноним 19/03/22 Суб 22:28:42 #139 №709015 
Если есть темная материя, то есть ли темная антиматерия?
Аноним 20/03/22 Вск 03:39:47 #140 №709025 
15528537859870.jpg
>>709015
А давай не так. Если тёмная материя, то светлая это антиматерия, мы живём в мире антиматерии и состоим из антиматерии.
Аноним 20/03/22 Вск 03:49:13 #141 №709026 
>>709015
Припоминаю, что были попытки поиска аннигиляции темных материи и антиматерии. Так что как минимум некоторые считают, что да.
Аноним 20/03/22 Вск 03:53:29 #142 №709027 
>>708895
Хуйня. Не верю в высокоразвитых агрессивных инопланетян. Мы прямо сейчас наблюдаем, что высокоразвитый и агрессивный - это скорее противоположности.
Аноним 20/03/22 Вск 04:09:53 #143 №709028 
>>709027
>Мы прямо сейчас наблюдаем, что высокоразвитый и агрессивный - это скорее противоположности.
Именно.
Аноним 20/03/22 Вск 05:44:42 #144 №709030 
>>709027
Может они настолько преисполнились в своей высокаразвитости что снос обитаемых планет под шоссе для них не геноцид а обычный вторник.
Аноним 20/03/22 Вск 10:24:58 #145 №709037 DELETED
>>708875
Ну два шара ЧД объединяются в 1 шар. С точки зрения геометрии часть пространства которое раньше было за горизонтом событий станет вне его. Если процесс не мгновенен то хер с ним дососет в центр материю с края, а если там сингулярность или объединение мгновенно то возникнут наверное некоторые парадоксы? Особенно на краю в пограничной области.
Аноним 20/03/22 Вск 12:37:45 #146 №709049 
>>709037
>С точки зрения геометрии часть пространства которое раньше было за горизонтом событий станет вне его.
Каким образом? Там не просто координаты х,у,з прямого пространства как в играх делается, оно же пиздец искажено.
У тебя пространство-время в больших масштабах у горизонта имеется, эээ блин, тяжело объяснять то что я и сам толком без матана и поллитры не могу.
Аноним 20/03/22 Вск 12:49:40 #147 №709052 
>>708995
Как успокоишь? Возьмешь в рот?
>>709015
Частицы темного вещества могут быть сами себе античастицами.
>>709037
>С точки зрения геометрии часть пространства которое раньше было за горизонтом событий станет вне его
С чего ты это взял?
Аноним 20/03/22 Вск 13:09:00 #148 №709053 
>>708895
Эта "догадка" хуйня на самом деле, годится только чтобы сай-фай писать. Во-первых, цивилизации, которая собралась уничтожать другие цивилизации, надо сначала убедиться, что её саму не уничтожит какой-нибудь галактический полицейский. Ведь если есть ты и ещё одна цивилизация в твоей галактике, то следует предполагать наличие других, и возможно множества других цивилизаций, в том числе более старых. Во-вторых, сидя у себя на квартале и пуляя какую-нибудь хуйню в другие системы, много не навоюешь. Уничтожишь родную планету - где гарантия, что у них нет колонии на другой планете? Уничтожишь все планеты - где гарантия, что у них нет орбитальных хабитатов или баз на мелкой хуйне, которую ты даже незаметил? Послать роботов? Теряешь фактор внезапности, даёшь возможность послать ответку или спрятаться. Самый надёжный способ - это расселить заранее роботов по всей галактике, которые будут уничтожать цивилизации превентивно, ещё до их выхода в космос. Но если бы такие роботы были, то они нас бы уже уничтожили, а мы всё ещё тут.
Аноним 20/03/22 Вск 13:09:51 #149 №709054 
>>709052
>Частицы темного вещества могут быть сами себе античастицами.
Можно объяснить для двачера?
Аноним 20/03/22 Вск 14:46:54 #150 №709061 
>>708996
Бамп вопросу.
Аноним 20/03/22 Вск 15:08:58 #151 №709062 DELETED
>>709052
Ну площадь равна пр^2, то есть при удвоении площади удвоение радиуса не произойдёт, то есть то, что было на границе радиуса обеих дыр в дальней точке от точки первого столкновения перестанет быть за горизонтом событий. Теоретически.
Аноним 20/03/22 Вск 17:22:30 #152 №709078 
16458303451110.jpg
У астероидов возможно своё магнитное поле?

А у лун планет? Я не про Ганимед, ибо его поле инициированное Юпитером, я имею в виду свои поля. Ну, или хотя бы инициированные у лун не гигантов, а у лун маленьких каменных планет.
Аноним 20/03/22 Вск 18:36:21 #153 №709085 
>>709078
Да.
Магнитное поле объектами создается либо токами, либо вращением/конвекцией проводника, либо ферромагнетизмом.
Последнее актуально для всех холодных объектов богатые железным минералами, но такие поля локальные и слабые.
Поля создаваемые токами могут возникает за счет обтеканием объекта плазмой или выбросам ионизованного вещества. Такие поля слишком не стабильные и больше характерны для комет или какого-нибудь Меркурия, но они могут служить источником намагничивания для предыдущего случая. Астероиды же формировалось тогда, когда Солнце еще не вышло на главную последовательность и постоянно плевалось плазмой, причем на порядке сильнее, чем во время теперешних вспышек. Попав астероид под такой выброс, он мог бы хорошенько намагнитить его.
Ну и вращение проводника это больше характерно для крупных объектов, где в недрах есть хорошо электропроводящий материал.
Аноним 20/03/22 Вск 22:01:12 #154 №709108 
>>707999 (OP)
Почему Земля крутится с Востока на Запад, а не наоборот? Почему не с Севера на Юг?
Аноним 21/03/22 Пнд 03:57:06 #155 №709137 
>>708996
Бамп вопросу
Аноним 21/03/22 Пнд 04:49:32 #156 №709139 
videoplayback.mp4
>>709108
Дык никто не знает, так получилось, при формировании нашей планетарной системы.
Там по разному всё может вращаться, очень много факторов, о которых человечество может ещё и не догадывается.
Аноним 21/03/22 Пнд 05:59:53 #157 №709141 
>>709108
Потому что север и юг и определены как точки на оси вращения Земли.
Аноним 21/03/22 Пнд 09:21:40 #158 №709146 
>>709052
Ты главное адрес свой говори, петушок. И не трясись так.
Аноним 21/03/22 Пнд 12:35:38 #159 №709164 
image.png
Можем ли мы в теории оценить наличие разумной жизни на экзопланетах путём оценки освещенности их тёмной стороны, например, во время транзита планеты по диску звезды или какими-то другими способами?

Таким образом мы сможем оценить наличие искусственного освещения инопланетных поселений.
Аноним 21/03/22 Пнд 13:55:38 #160 №709167 
>>709164
Не можем, потому что источники такого света слишком слабы.
Аноним 21/03/22 Пнд 14:02:38 #161 №709169 
>>709164
Нет, мы едва различаем наличие планет, про их характеристики говорим лишь просто по их орбитам.
Аноним 21/03/22 Пнд 14:04:04 #162 №709171 
>>709054
Частицы темного вещества могут аннигилировать при столкновении друг с другом.
>>709062
>перестанет быть за горизонтом событий.
Не перестанет.
>>709146
Ну хорошо, чушок, записывай. Москва, улица Пушкина, дом Колотушкина, квартира - та, в которой я твою маму ебал, ты должен помнить.
Аноним 21/03/22 Пнд 14:06:12 #163 №709172 
>>709171
>Частицы темного вещества могут аннигилировать при столкновении друг с другом.
А мы бы тогда такое замечали по идее?
Аноним 21/03/22 Пнд 14:06:55 #164 №709173 
>>709171
>манька зассала всю табуретку и слилась
Ясное дело, это же не на харкаче кукарекать, сидя на уроке.
Аноним 21/03/22 Пнд 15:18:41 #165 №709183 
>>709172
Да, могли бы. Но аннигиляция темной материи не обязательно может рождать именно фотоны. Вполне может рождаться пара нейтрино-антинейтрино, которые тяжело детектит. Или пара электрон-позитрон, которые тяжело отделять от общего фона.
В добавок темная материя не концентрируется как барионная материя, что делает ее аннигиляцией редким явлением самом по себе.
Аноним 21/03/22 Пнд 17:00:07 #166 №709203 
может ли космонавт угнать каробль в открытый космос? или в мкс врезаться?
Аноним 21/03/22 Пнд 17:03:40 #167 №709205 
>>709203
>в открытый космос?
Дельты не хватит.

>в мкс врезаться?
Может.
Аноним 21/03/22 Пнд 19:39:25 #168 №709233 
>>708922
А как ИИ поможет?
Аноним 21/03/22 Пнд 19:41:45 #169 №709234 
>>708808
бампь
Аноним 21/03/22 Пнд 20:49:41 #170 №709245 
>>709233
Рабочий ИИ (Интеллектуальные системы, а маняхуйня фантастов) может делать все, что мясной мешок, но с минимум затрат, в добавок ИИ может работать в очень различных условиях. Для функционирования мясного мешка нужна весьма ресурсно затратная система жизнеобеспечения, и если что мясной мешок быстро выходит из строя.

Чтоб строить мегаконструкции нужно имеет инфраструктуру по сбору и транспортировки энергии и материалов, а еще производством прям на месте. Вся это хуйита должно управляться и обслуживаться НА МЕСТЕ с минимальными задержками. Тут кучей мясный мешков не обойдешься.
Аноним 21/03/22 Пнд 22:22:23 #171 №709252 
>>709245
Вопрос в том, нужны ли мегаконструкции в принципе?
Чисто теоретически, зачем они могут пригодиться? Разве что делать внепланетарное поселение?
Аноним 21/03/22 Пнд 22:25:30 #172 №709254 
Знаете почему во всех фильмах инопланетяне постоянно показаны как гуманоиды?

Потому что все они произошли от одного прото-вида, который в далёком прошлом разослал корабли-поселения во все стороны галактики.
И спустя миллионы лет население, флора и фауна всех миров развивались независимо, отчего и возникли все эти кучи гуманоидов, которые выглядят по-разному, но все ещё имеют башку, 2 руки и 2 ноги.
Аноним 21/03/22 Пнд 22:29:13 #173 №709255 
>>709252
> Чисто теоретически, зачем они могут пригодиться? Разве что делать внепланетарное поселение?
Эксперименты со сверхвысокими энергиями, черными дырами и т.п. Ещё как релятивистское оружие.
Аноним 21/03/22 Пнд 22:42:54 #174 №709258 
>>709203
Нет не может
Аноним 21/03/22 Пнд 22:56:37 #175 №709261 
>>709255
>Эксперименты со сверхвысокими энергиями
Для этого нужны объекты размером больше солнечной системы. Это тебе не сраная луна, тут уже другое.
Аноним 22/03/22 Втр 05:08:47 #176 №709286 
>>708996
Бамп вопросу?
Аноним 22/03/22 Втр 05:18:46 #177 №709287 
Долго ли Белку и Стрелку дрессировали управлять ракетой?
Аноним 22/03/22 Втр 06:49:06 #178 №709289 
>>709287
Нет. Они ракетой не управляли.
Аноним 22/03/22 Втр 10:53:39 #179 №709294 
>>709252
Мегаконструкции это очень полезная вещь в народном хозяйстве.
В первую очередь это сбор энергии от Солнца и передача туда, где с ней проблемы. Энергия всегда нужна.
Ну и второе это колонизация или промышленное освоение адских миров типа Меркурия и Венеры.
И третье это транспортная инфраструктура. Целая сеть мегахуевин позволить эффективно перекидоваться грузами по всей солнечной системе.
Аноним 22/03/22 Втр 13:38:39 #180 №709295 
>>709172
Да, по гамма-квантам. Проводятся регулярные наблюдения на предмет этого гамма-излучения. Иногда регистрируются интересные сигналы, которые могут быть связаны как раз с аннигиляцией частиц темного вещества.
>>709173
Ну и зачем ты кукарекаешь на харкаче, сидя на уроке, чухан? А что если тебя запалит училка и скалкой в жопу выебет за такую хуйню?
Аноним 22/03/22 Втр 14:25:09 #181 №709298 
>>708996
Очень вряд ли.
Комплексная геометрия не имеет аналогов в привычной нам трехмерной геометрии. Даже ебучий спинор вызывает боль у аспирантов, а без него никак не объяснить калибровочные поля и связанные с ним симметрии и их нарушения.
Аноним 23/03/22 Срд 13:43:00 #182 №709399 
>>709254
Ковергентная эволюция.
Аноним 23/03/22 Срд 13:46:00 #183 №709400 
>>709295
>запалит училка и скалкой в жопу выебет
Соболезную, что такое с тобой приключилось. Именно это стало причиной твоего вылета из школы для особенных и прогрессирующей тупости?
Аноним 23/03/22 Срд 14:13:19 #184 №709401 
>>709254
>Знаете почему во всех фильмах инопланетяне постоянно показаны как гуманоиды?
Знаю: меньше затрат на костюмы.
Аноним 23/03/22 Срд 23:37:44 #185 №709453 
Тупой вопрос: если разгонять реактивную струю на выходе в сопле каким-нибудь йоба-полем ракета полетит быстрее?
Аноним 24/03/22 Чтв 01:04:14 #186 №709460 
>>709401
Как у вас там в 1990-м?
Какие нахуй костюмы? попустись
Аноним 24/03/22 Чтв 10:11:45 #187 №709467 
>>709453
Таки да, но есть много подходных камней. В первую очередь у химических двигателей выхлоп очень слабо ионизован, газы будет слабо реагировать на поля. Во вторых установка создающая поля довольно массивная, да еще требует много энергии на работу.
Аноним 24/03/22 Чтв 10:26:40 #188 №709472 
>>709453
Если ты попробуешь для этого батарейки использовать, то ты лишь потеряешь дельту из-за массы батарей, они менее емкие чем топливо которое тащишь, проще больше топляка на борт взять.
Если попробуешь химический источник... то то же самое, ты и так его на борту держишь, проще и надежнее залить больше соляры.
И тут есть что-то на порядки более энергоемкое чем химия - ядрены реакторы. Если ты умудришься удобно запаковывать, охлаждать его и гонять им топляк, то тебе и химия не особо нужна.
Аноним 24/03/22 Чтв 11:46:49 #189 №709481 
>>709460
Гуманоидные алиены появились в кино не сейчас, а сто лет назад, и стали стандартным клише в сериалах типа стар трека. В фильмах, готовых тратить больше денег на дизайн и разработку костюмов или эффектов, алиены бывали и негуманоидными (Чужой, Звёздный Десант и т.п.)
Аноним 24/03/22 Чтв 11:59:53 #190 №709482 
>>709481
Чужой гуманоид же. Башка сверху, две руки, две ноги, ну и хвост впридачу, что никогда не мешало гуманоидам быть гуманоидами.
Вот где прям классно оригинально запилили - Blindsight.
Аноним 24/03/22 Чтв 12:36:05 #191 №709487 
>>709400
Лол, как же тебе больно. Вазелина дать, чтоб болело меньше?
Аноним 24/03/22 Чтв 23:00:51 #192 №709513 
>>708091
Запросто. У угля абиогенное происхождение.
Аноним 24/03/22 Чтв 23:13:50 #193 №709515 
>>708227
На сайте https://minorplanetcenter.net прям на главной есть табличка Close Approaches - это как раз всякие каменюки которые пролетали, пролетают и будут пролетать мимо Земляшки в ближайшее время, не все конечно, а только те которых повезло когда либо засечь. Там указаны название каменюки, дата и время наиболее тесного сближения, минимальное расстояние в радиусах орбиты Луны, то есть если меньше 1, то каменюка пронесется внутри орбиты Луняши, страшно аж, ну и размер каменюк там указан еще в метрах. Короче, летает много всякого. Можно начинать бояться прямо сейчас.
Аноним 24/03/22 Чтв 23:44:41 #194 №709517 
>>708632
Во вселенной есть равновесие. Хаос не нарастает. Его дохуя, но примерно столько же сколько задано было при начальных условиях симуляции. Звезды и всякие квазары это маленький пук из того что есть, считай что это легкие упорядоченные флуктуации в океане хаоса.
Аноним 24/03/22 Чтв 23:49:08 #195 №709518 
>>708704
Кто обладает знанием, тот обладает властью. Власть слишком лакомый кусочек, чтобы делиться ей со всеми. Естественно приходится дезинформировать потенцивальных врагов, чтобы они не развились сильнее чем их можно контролировать.
Аноним 25/03/22 Птн 00:39:53 #196 №709519 
>>709517
>Его дохуя
Говори прямо - Тёмная Материя. Форма материи, не участвующая в электромагнитном взаимодействии и поэтому недоступная прямому наблюдению. Составляет порядка четверти массы-энергии Вселенной и проявляется только в гравитационном взаимодействии.
Аноним 25/03/22 Птн 12:07:45 #197 №709563 
>>709519
Шизик, таблетки.
Аноним 26/03/22 Суб 01:14:21 #198 №709614 
>>709519
>Составляет порядка четверти массы-энергии Вселенной
Как посчитали?
Аноним 26/03/22 Суб 13:34:31 #199 №709642 
>>709614
По модам реликтового излучения.
Аноним 26/03/22 Суб 17:08:19 #200 №709660 
Откуда инфа, что катаклизм, который случился с Венерой, сделав её тем, что что она есть сейчас, а не вторую Землю с температурами от 20 до 50 по цельсию, произошла примерно 300 миллионов лет назад? Почему не 150? Не 50? Не 1 лям лет назад? Или почему не лярд лет назад? Как это определяется?
А то это похоже на уровень "прикинуть на глаз, примерно так прочувствовать"
Аноним 26/03/22 Суб 20:28:06 #201 №709686 
>>709660
Возраст поверхности планеты оценивается по количеству ударных кратеров, соответственно так и посчитали.
Аноним 27/03/22 Вск 00:11:01 #202 №709696 
Простите за тупой вопрос.
Из солнечной системы летит в сторону системы альфа центавра пренебрежимо малая масса со скоростью 0.1 от световой стартовой скорости. В связи с приближением к системе альфа центавра тело искривляет свою траекторию под действием силы тяжести.
1 Вопрос: какой максимальный стартовый угол отклонения допустИм для то чтобы тело соприкоснулось с диском фотосферы одного из компонентов системы.
2 Вопрос, который самый важный: Как это рассчитать?
Аноним 27/03/22 Вск 06:57:31 #203 №709706 
>>709660
>А то это похоже на уровень "прикинуть на глаз, примерно так прочувствовать"
С точки зрения малолетнего дебила, безусловно.
Погугли, как вычисляются возрасты планет.
Аноним 27/03/22 Вск 08:49:47 #204 №709710 
5465466546.jpg
65756675756.png
Это точно не рофлянный снимок? очертания реал солнечного чувака который орет и шлет всех нахер

>Фотографии создал европейский зонд Solar Orbiter, разработанный для изучения звезды. Космический аппарат сделал снимки 7 марта 2022 года, находясь на одинаковом расстоянии от Земли и Солнца — 75 миллионов километров.
Аноним 27/03/22 Вск 10:20:43 #205 №709715 
>>709710
https://ru.wikipedia.org/wiki/Парейдолия
Аноним 27/03/22 Вск 10:39:33 #206 №709717 
>>709696
У ЛЛ в первом томе есть типовая задача, которая разбирает твой вопрос.
Аноним 27/03/22 Вск 12:10:54 #207 №709721 
1646634748465.jpg
Вот мы обнаружили, значит-с, астероид. Летит он себе там где-то за Сатурном по своей орбите и никому не мешает.
А мы его сидим на Земле и меряем.
Откуда мы знаем, что перед нами астероид, равномерно распределивший массу по объёму аки кусок пемзы? Может, это толстая корочка вокруг пустого ничего, как пустая коробка из под сока? А может, там тонкая корочка и немножечко ядра, а между ними едва ощутимая газопылевая масса?

Как вообще происходит сканирование подобных тел?
А крупных планемо и планет?
Вообще, мы ж нихуя сканировтаь не умеем, получается. Даже нашу родную Землю мы не умеем сканировать больше, чем на пару десятков км, а уже про другим планеты выводе делаем
Аноним 27/03/22 Вск 12:48:34 #208 №709724 
>>709721
А мы и не знаем. Потому и посылают миссии к астероидам, чтобы их изучить. Но полый внутри астероид вряд ли возможен, его бы сдавило гравитацией. Разве что металлический, но тогда вопрос, как образовалась полость?
>А может, там тонкая корочка и немножечко ядра, а между ними едва ощутимая газопылевая масса?
А это тем более невозможно, с чего бы ядру быть в середине? Оно бы столкнулось со стенкой и проломило её.

>Даже нашу родную Землю мы не умеем сканировать больше, чем на пару десятков км
Землю мы сканируем сейсмографами, по прохождению звуковой волны через толщу Земли. А массу Земли знаем по движению луны и спутников.
Аноним 27/03/22 Вск 13:51:02 #209 №709728 
>>709717
Благодарю.
>>709724
>Но полый внутри астероид вряд ли возможен
Ну-уу. Есть смелые предположения что там есть крупные пустоты. Ну не так конечно как у Жуль Верна, но..
Аноним 27/03/22 Вск 16:03:24 #210 №709743 DELETED
>>709724
>Но полый внутри астероид вряд ли возможен, его бы сдавило гравитацией
Да ты, я смотрю, последние пять лет в анабиозе лежал
Аноним 27/03/22 Вск 18:18:49 #211 №709748 
>>709724
Простой сценарий образование полого астероида.
Комета сталкивался с рандомной мелкой каменюкой, после удара вся силикатная пыль медленной оседает на поверхность льда.
Комета мигрируют во внутрь системы, где ее начинает жарить Солнце, выветривать солнечный ветер и оседать пыль.
Пыль на границе цементируется за счет медленного испарения льда и действия солнечного ветра, формируя этакую жесткую скорлупу. Лед же в центре медленно выкипает за миллионы лет, оставляя полость.
Аноним 28/03/22 Пнд 01:09:41 #212 №709781 
1648418980145.png
Разгадка парадокса Ферми — это задача трёх тел. Вот он этот великий фильтр.
Это же очевидно. В определенный момент случайные флуктуации гравитационного поля приводят к тому, что обитаемая планета сходит со "стабильной" орбиты и жизнь на ней умирает
Аноним 28/03/22 Пнд 01:53:46 #213 №709782 
>>709642
Каким ещё модам?
Аноним 28/03/22 Пнд 09:16:40 #214 №709790 
>>709706
Чмо, ты немножко жопой вопрос читало.
Аноним 28/03/22 Пнд 09:28:16 #215 №709791 
1627372455258.jpg
>>709781
> великий фильтр
Недавно пересматривал мини-сериал Конец детства. И неожиданно для себя понял, что сериал про Великий фильтр. Некая супер развитая цивилизация уничтожает другие цивилизации, которые могли бы развиться до его уровня. При этом они оставили в живых цивилизацию краснокожих, потому что они достигли предела развития и не представляют угрозы.
Аноним 28/03/22 Пнд 12:10:45 #216 №709801 
>>709782
Мода поляризации и спектр мощности, который имеет отклонения от теплового.
Аноним 28/03/22 Пнд 17:18:48 #217 №709817 
>>707999 (OP)
почему на кометы и астероиды с вытянутой орбитой не отправляют научные аппараты ? это же практически бесплатный способ долететь до границ солнечной системы
Аноним 28/03/22 Пнд 17:28:35 #218 №709818 
>>709817
Каким образом бесплатный? Аппарат будет ловить пролетающую мимо на большой скорости комету арканом?
Аноним 29/03/22 Втр 12:57:03 #219 №709854 
>>709791
>мини-сериал Конец детства
Это же какое-то лютое говно со спиритуализмом и антинаучностью. Меня при первом просмотре чуть не вывернуло.
Аноним 29/03/22 Втр 13:03:23 #220 №709855 
>>709817
Чтобы "отправиться" на астероид аппарату нужно выйти на ТУ ЖЕ САМУЮ ОРБИТУ, что и астероид т.е. никакой бесплатности тут нет.

Если аппарат может выйти на орбиту "до границ солнечной системы" - он и без астероида выйдет. А если не может, то и с астероидом он не встретится (а если встретится, то они просто пронесутся мимо друг-друга на ЙОБА-скоростях без какого-либо эффекта или пользы вообще).
Аноним 29/03/22 Втр 13:34:11 #221 №709856 
А есть на двачике где-то тред картографии, любителей карт, рельефов, регионов, такого всего?
В саентаче не заметил, там и игнрируют этот вопрос, где еще искать не знаю.
Аноним 29/03/22 Втр 14:09:34 #222 №709860 DELETED
>>709856
Создай, я люблю всякие карты.
Аноним 29/03/22 Втр 14:23:53 #223 №709862 
image.png
А есть на двачике где-то тред картографии, любителей карт, рельефов, регионов, такого всего?
В саентаче не заметил, там и игнрируют этот вопрос, где еще искать не знаю.
Аноним 29/03/22 Втр 14:39:14 #224 №709863 
>>707999 (OP)
Что можно установить на Луне, чтобы было видно с Земли невооруженным глазом? Можно ли использовать для этого гигантскую светодиодную ленту, или светильник?
Аноним 29/03/22 Втр 14:52:06 #225 №709864 
>>709863
С современными технологиями - ничего.
Надо гигаваттные мощности и светить направленно, чтобы как-то это выделялось либо на фоне дневного неба, либо на фоне освещенной солнцем поверхности.
Аноним 29/03/22 Втр 15:16:12 #226 №709865 
>>709864
А для того что бы было видно хотя бы что то на фоне ночного неба?
Аноним 29/03/22 Втр 15:19:19 #227 №709866 
>>709863
>Что можно установить на Луне, чтобы было видно с Земли невооруженным глазом?
Каменный хуй размером с лунное море.
Аноним 29/03/22 Втр 15:21:41 #228 №709867 
>>709865
На фоне ночного неба... Луна и так прекрасно видна потому что освещена солнцем.
Тебе надо перебить свет солнца, примерно 300 ватт света с квадратного метра. Звучит немного, но у тебя закон обратных квадратов.
Гигаватты надо фигачить.
Аноним 29/03/22 Втр 15:54:43 #229 №709870 DELETED
>>709863
Настелить фольги пару квадратных километров, думаю будет видно как светлое пятнышко.
Аноним 29/03/22 Втр 15:56:44 #230 №709872 
Если магическим образом научится передавать сообщение быстрее света, то каким образом из этого следует, что мы можем отправлять сообщения в прошлое?
Аноним 29/03/22 Втр 16:02:52 #231 №709873 
>>709867
Он имеет в виду в новолуние наверное.
Аноним 29/03/22 Втр 16:09:53 #232 №709875 
>>709873
Да, я имел ввиду неосвещенную часть Луны, она же не освещается Солнцем
Аноним 29/03/22 Втр 16:42:06 #233 №709880 
>>709872
Никаким, уже разбирали этот бред тут.
Аноним 29/03/22 Втр 17:01:25 #234 №709883 
>>709873
>>709875
А я упоминал в прошлом посту что на дневном небе надо перебить свет неба.

Можно заморочиться и посчитать сколько ватт и люменов надо, но надо для этого дать какой-то критерий видимости.
Аноним 29/03/22 Втр 23:59:33 #235 №709899 
>>709872
https://en.wikipedia.org/wiki/Tachyonic_antitelephone

>>709880
Кто разбирал, где?
Аноним 30/03/22 Срд 12:42:48 #236 №709911 
1648633364138.jpg
Че там за астероид опять летит? Говорят на этот раз точно пиздарики. Спейсач успокой меня или отправь готовиться к вечности.
Аноним 30/03/22 Срд 13:13:00 #237 №709913 
>>709911
>или отправь готовиться к вечности
А без астероида ты типа не готов? В любой момент может кирпич на голову упасть, или машина собьёт. Вероятность во много раз выше астероида.
Аноним 30/03/22 Срд 14:44:50 #238 №709917 
>>709916
Если не нравится кирпич, то давай так.
У тебя больше шансов подохнуть от спидорака вызванным космическим мюоном, нежели чем от падения астероида.
Аноним 30/03/22 Срд 15:05:01 #239 №709921 
>>709899
Ищи по архиву этого треда.
Аноним 30/03/22 Срд 15:05:57 #240 №709922 
image.png
>>709911
Апофис летит. Как летел так и летит, уже почти 20 лет ждем. По земляшке не попадет, поэтому всем похую.
Только он и будет ближайшее мимопролетающее тело в 2029, остальные сильно дальше.
Аноним 30/03/22 Срд 15:38:47 #241 №709926 
http://tropic.ssec.wisc.edu/real-time/mimic-tpw/global/main.html
Почему они показывают только над водой? Наоборот над сушей же важно знать сколько воды, чтоб потопы считать и все такое.
Аноним 30/03/22 Срд 15:44:08 #242 №709927 
image.png
>>709863
Почти рилейтед: https://what-if.xkcd.com/13/
Аноним 30/03/22 Срд 15:57:23 #243 №709929 
1648645037147.jpg
>>709922
Аноним 30/03/22 Срд 16:17:50 #244 №709931 
Как выглядели материки когда упал астероид убивший динозавров?
Аноним 30/03/22 Срд 16:19:59 #245 №709933 
tumblro6slxtsPaS1rasnq9o11280.jpg
>>709931
Аноним 30/03/22 Срд 19:35:25 #246 №709940 
>>709922
>По новым данным Апофис приблизится к Земле в 2029 году на расстояние 38 400 км от центра Земли (по другим данным: 36 830 км, 37 540 км, 37 617 км)
А это не слишком уж близко?, при том что Эффект Ярковского еще может подкорректировать его прям в землю
Аноним 30/03/22 Срд 19:56:42 #247 №709941 
inarticlea9723bc2a4.png
>>709933
Аноним 30/03/22 Срд 20:04:29 #248 №709942 
>>709940
Год назад JPL свежие цифры подвезли, 100 лет еще гипофиз по сраной земляшке не сможет попасть, траекторию до считанных километров уточнили.
Жди когда Йеллоустоун перданет или нюки расчехлят, космос пока нас не собирается уб
Аноним 30/03/22 Срд 20:06:19 #249 №709943 
>>708947
Ну такой себе довод. Не факт, что тот кто нашумел в эфире, сможет принять меры для защиты против повторного пристрелянного залпа. Да и не будут вовсе стрелять, например, а устроят другой какой-нибудь катаклизм, от природы которого жертва ни то что бы смогла защититься, а и вовсе не осознала бы ее.
Аноним 30/03/22 Срд 22:51:38 #250 №709945 
>>708895
https://www.youtube.com/watch?v=xAUJYP8tnRE
Аноним 31/03/22 Чтв 00:17:17 #251 №709949 
>>709921
Лень искать. Но в общем, слать надо таких разбирателей, хули тред засирают.
Аноним 31/03/22 Чтв 02:52:29 #252 №709956 
2.jpg
>>709940
>Апофис приблизится к Земле в 2029 году
А тут и не будет уже никого живого.
Аноним 31/03/22 Чтв 03:28:44 #253 №709957 
>>709956
Ах, если бы. Вы все обещаете, а я так до сих пор не помер.
Аноним 31/03/22 Чтв 14:02:55 #254 №709982 
>>709943
Пример такого катаклизма можешь привести?
Аноним 31/03/22 Чтв 14:42:50 #255 №709984 
>>709982
> а и вовсе не осознала бы ее.
> Пример такого катаклизма можешь привести?
> НЕ ОСОЗНАЛА БЫ
> ПРИМЕР
Ты альтернативно одаренный, чи шо?
Аноним 31/03/22 Чтв 14:44:54 #256 №709985 
>>709984
Маняфантазёр, успокойся.
Аноним 01/04/22 Птн 01:48:28 #257 №710013 
cern-pic700-700x467-51273.jpg
Если вселенная предельно фаталистична, значит всё что должно произойти в будущем — уже произошло, а если что-то уже произошло, значит это уже прошлое.

В чем я не прав?
Аноним 01/04/22 Птн 02:06:15 #258 №710015 
>>710013
Нет. Тогда нет понятий "прошлое" и "будущее", они бессмысленны. Всё просто есть.
Аноним 01/04/22 Птн 14:08:50 #259 №710030 
>>710013
>Если вселенная предельно фаталистична
Она вероятностна, а не фаталистична, лол.
Аноним 01/04/22 Птн 17:29:26 #260 №710036 
image.png
image.png
image.png
Есть что-то похожее на Valles Marineris на других телах?
Какие есть гипотезы такой ебанутой формации?
Аноним 01/04/22 Птн 22:13:17 #261 №710053 
>>709855
А, точно. где здесь ближайший биореактор?
Аноним 01/04/22 Птн 22:18:04 #262 №710055 
Вояджеры прошли гелиосферу?
Аноним 01/04/22 Птн 22:42:16 #263 №710061 
>>710055
Несколько раз.
И пройдут еще не раз.
Аноним 01/04/22 Птн 22:43:10 #264 №710062 
>>710036
>Какие есть гипотезы такой ебанутой формации?
Я лично много думал над этим планетарным шрамом. наиболее логичным мне показалась идея, что на Марс что-то упало (может быть, третий спутник, который был не камушком на 10км типа Деймоса-Фобоса, а чем-то существенным), из-за чего из-за натяжения/упругости/вязкости/тягучести и прочих факторов поверхности в совокупности в момент удара это-то чего-то по Марсу, Марс треснул своей поверхностью. Логично, что если Маринер реально результат столкновения, то должно быть место столкновения, и, если оно так, то гора Олимп - это нихуя не вулкан, а развалившаяся луна или астероид, который упал на Марс и вызвал такую трещину дальше по поверхности по ходу направления приложения сил инцерции.
Но даже если бы это было так, это не могло бы никак вообще объяснить такую ширину и протяжённости. Даже если бы Марс был покрыт монолитной корочкой подобно скорлупе, покрывающей яйцо, ударная трещина могла бы быть длинной, но не могла бы быть такой широкой.
Отсюда напрашивает вывод (домысел): данная формация искуственная. Кем она делалась - вообще поебать, важно понять зачем и какой в ней смысл.
Аноним 01/04/22 Птн 22:49:15 #265 №710063 
>>710062
А если выпало не целиком, а градом распидорашенных ошметков разлившимся в пределе Роша объектом? Протяженность намекае.
Только было бы более кратеровидно, а это глубинный каньон, точнее система.
>Отсюда напрашивает вывод (домысел): данная формация искуственная. Кем она делалась - вообще поебать, важно понять зачем и какой в ней смысл.
Ты предполагаешь, что эта формация делалась, и отсюда недопонимание как и зачем, а что если формация была даже не близко такая, и то что мы видим сейчас - результат эрозии и эпох запустения, тогда где-то в глубинах может быть лежат артефакты, заваленные опоры, конструкции, которые бы подсказали о предназначении этого места.

>>710036
А рядом лавовых трубок не обнаружено, кстати? Поселить там огурцов в трубке сычевать, будут роверами кататься по ущельям и собирать всякое говно.
Аноним 01/04/22 Птн 22:56:42 #266 №710065 
>>710063
>А если выпало не целиком, а градом распидорашенных ошметков разлившимся в пределе Роша объектом?
Нет. Открой карту Марса и посмотри на эту формацию. Это не горный хребет по экватору, а углубление с отвесными стенами. Больше похоже на то, что рылись широченные пути между определённым долинами/низинами по кротчайшему расстоянию - прямой. Может, вообще пыталсь провести ров для того, чтобы вся вода с северного океана утекла под гору (при условии, что у пещер и у Лабиринта Ночи глубина Марина ниже, чем у приокеанских мест).
А так больше похоже, как будто циркуляркой проехались по кирпичу, только сперва на пару сантиков проебались и пробным заходом чутка выше пильнули и только потом где надо. Ну то есть вообще на результат падения осколков с предела Роша не похоже.
Аноним 01/04/22 Птн 23:11:07 #267 №710067 
Думал о двойных звёздах, потом мысль соскользнула в рассуждения о существовании двойных астероидов (сам астероид и его спутник), и подумал о двойных планетах.
Если есть две разномассивные звезды, то есть и разномассивные двойные планеты: Земля-Луна, Плутон-Харон. А что делать, если звёзды примерно равномассивные, но разные? Если есть пара звёзд, где одна звезда маленькая старая красная звезда, а её компаньон - молодая и шутливая горящая белая звезда, то могут ли быть аналогичные пару у двойной планеты?
Например, Как-Марс и Как-Нептун вращаются друг вокруг друга?
Из примера выше вытекает ещё один вопрос: а если у такой двойной планеты появится спутник или ряд спутников, то будут ли эти спутники вращаться вокруг общего центра масс двойной планеты и как на них и их орбиту повлияет вечный танец из приближения-отдаления компонентов двойной планеты?
Аноним 01/04/22 Птн 23:24:07 #268 №710069 
image.png
>>710065
Кстати, хорошо подметил.
Выглядит будто каналы ведущие под Фарсиду, даже возвышенности будто продолжают там.
Ух бля, интрига как в сосничестве.

А есть что на эту тему? Кинцо не сильно ебанутое? Игоры? В редфакшенах раскрывали вопрос?
Аноним 01/04/22 Птн 23:33:12 #269 №710070 
>>710061
> Несколько раз.
Т.е не прошли? Я просто постоянно читаю что они что-то там прошли а потом пишут что нет, кому верить?
Аноним 01/04/22 Птн 23:44:11 #270 №710071 
>>710070
10 лет назад прошли, резкое падение зафиксировали.
Аноним 02/04/22 Суб 02:27:08 #271 №710079 
ponchiki.jpg
>>710036
Вы когда-нибудь пончики в масло кидали? Они там сначала меньше объёмом, а потом раздуваются и трескаются. Думаю с планетами такая же херь. На начальном этапе они меньше размерами, а потом увеличиваются и получаются всякие разломы в коре.
Аноним 02/04/22 Суб 03:07:13 #272 №710084 
>>710079
В планетарном масштабе наоборот, анон. Планеты остывают и уменьшаются по понятным (надеюсь) причинам.
То что ты написал я не видел в планетологии. Планеты не нагреваются и не увеличиваются спонтанно.

Но мысль может быть такая, дай-ка угадаю, что на самом деле это был огромный импактор в Элладе или на севере, что вызвал и трещины Маринера, и вулканы Фарсиды? Мне нравится такая гипотеза.
Интересно, что аноны про это скажут.
Аноним 02/04/22 Суб 07:18:43 #273 №710086 
>>710069
>А есть что на эту тему? Кинцо не сильно ебанутое? Игоры? В редфакшенах раскрывали вопрос?
Ничего, это мои домыслы, основанные сугубо на созерцании фото рельефа и карт Марса, а я всего лишь писатель, который пока, кроме одной комедийной книги, даже близко не сайфайной, не написал.
Аноним 02/04/22 Суб 07:49:17 #274 №710087 
16445454161491.jpg
Гелиосинхронные орбиты - это всегда орбиты, проходящие через полюс, или они могут быть и экваториальными?
Аноним 02/04/22 Суб 09:44:39 #275 №710091 
>>710087
Эээ... Гелиосинхронные не проходят через полюс. Иначе бы они съезжали.
Да, по идее ты можешь быть технически солнечно-синхронным и на другой орбите, зависит от точки поверхности относительно которой ты синхронен.
Аноним 02/04/22 Суб 09:50:24 #276 №710092 
>>710091
>Да, по идее ты можешь быть технически солнечно-синхронным и на другой орбите, зависит от точки поверхности относительно которой ты синхронен.
Не понял. Покажи примеры для визуального восприятия.
Аноним 02/04/22 Суб 11:02:10 #277 №710095 
>>710092
Надо огурцов запускать.
На слово не поверишь?
По идее надо сперва уточнить терминологию. Что ты подразумеваешь под гелиосинхронной орбитой?
Аноним 02/04/22 Суб 11:25:51 #278 №710097 
>>710095
>На слово не поверишь?
Я не понимаю, а не не верю.
Аноним 02/04/22 Суб 16:28:09 #279 №710113 
>>707999 (OP)
Каким образом тяжелые элементы оказываются
1.За пределами того что остается от ядра сверхновой?
2. За пределами центров звезд следующего поколения (которые формируются из получившихся в результате взрыва сверхновых газопылевых туманностей)?
3. За пределами ядер протопланентых образований?
4. Каким образом эти тяжелые элементы оказываются не распылеными на отдельные молекулы, а крупными образованиями, например золотыми самородками?
Аноним 02/04/22 Суб 16:42:07 #280 №710117 
>>710097
Дай мне еще время, братюнь, я накидался в нижнюю околоземную.
Потом тебе без проблем орбит накидаю.
Ты лучше скажи ответ на главный вопрос:
ЧТО ТЫ ПОДРАЗУМЕВАЕШЬ ПОД ГЕЛИОСИНХРОННОЙ ОРБИТОЙ
и я (или кто-то еще) тебе исполню (исполнит)
Аноним 02/04/22 Суб 16:46:47 #281 №710119 
>>710113
1. Сверхновая это эпически катаклизмическое событие, которое выбрасывает огромное количество энергии.
Извиняюсь, я не дал понять, ОГРОМНОЕ количество энергии.
Извиняюсь, ОООГРРРООООООООМНОООООООООЕЕЕЕЕЕ количество энергии.
Как бы ты ни прикидывал цифры если ты делаешь расчеты по сверхновым - цифры больше.
ООООООООООООООГГГГГГГГГГГГГГГГРРРРРРРРРРРРРООООООООООООООМММММММММММНННННННННОООООООООООЕЕЕЕЕЕЕЕЕЕ количество энергии.
Понятно что осколкам атомов не только прилетят протоны и нейтроны, но еще и пиздюли чтобы те летели куда подальше.

3 см 1
3 см 1
4 А они оказываются. Большинство из них и распидорашивает в ноль, выживают только те что живут дольше всех, их ты и наблюдаешь.

Надеюсь, помогает теб...

Нет,
ОООООООООООООООООООГГГГГГГГГГГГГГГГГРРРРРРРРРРРРРРРРООООООООООООООООММММММММММММММММНННННННННННННННННЫЫЫЫЫЫЫЫЫЫЫЫЫЫЫЫЫЕЕЕЕЕЕЕЕЕЕЕЕЕЕЕЕ ЦЦЦЦЦЦЦЦЦЦЦЦЦЦЦЦЦЦЦЦЦЦЦЦЦИИИИИИИИИИИИИИИФФФФФФФФФФФФФФФФРРРРРРРРРРРРРРРРРРРРРРЫЫЫЫЫЫЫЫЫЫЫЫЫЫЫЫЫЫ у сверхновых.

Тебе даже в килопарсеке не захочется быть, настолько это адъ.
Аноним 02/04/22 Суб 18:09:12 #282 №710123 
>>710117
>ЧТО ТЫ ПОДРАЗУМЕВАЕШЬ ПОД ГЕЛИОСИНХРОННОЙ ОРБИТОЙ
Из википедии
>Солнечно-синхронная орбита (иногда именуемая гелиосинхронной[1]) — геоцентрическая орбита с такими параметрами, что объект, находящийся на ней, проходит над любой точкой земной поверхности приблизительно в одно и то же местное солнечное время. Таким образом, угол освещения земной поверхности будет приблизительно одинаковым на всех проходах спутника.
Но при этом видос в статье показывает полярную орбиту. И что-то нихуя не видно, чтобы спутник пролетал в одно и то же время одну и ту же точку пространства
И уж тем более, я не понимаю, что это за орбита такая, когда спутник "висит" в одной и той же точке неба. Например, "вешаем" спутник на орбиту так, чтобы между центром Земли и самим спутником выстраивалась прямая до каких-нибудь плеяд, а рядом на той же высоте вешаем спутник, который выстаривает прямую до Поллукса. Они же не может висеть на одной орбите, они же должны как-то пересекаться и/или не находиться там всегда и/или одно и то же время
Аноним 02/04/22 Суб 21:01:57 #283 №710131 
>>710067
>есть пара звёзд, где одна звезда маленькая старая красная звезда, а её компаньон - молодая и шутливая горящая белая звезда
Двойные звёзды образуются в олно время и они одного возраста; захват одной звезды другой - слишком редкое событие.
>если у такой двойной планеты появится спутник или ряд спутников, то будут ли эти спутники вращаться вокруг общего центра масс двойной планеты
Это задача трёх+ тел, она пока ее решена.
Аноним 02/04/22 Суб 21:33:03 #284 №710133 
>>710123
Там не полярная орбита.
Аноним 03/04/22 Вск 15:09:29 #285 №710181 
image.png
Ну и в чем он не прав.
Аноним 03/04/22 Вск 16:35:33 #286 №710186 
>>710181
Тем что он постит в телеграммчике, а не служит. Заметь как он пишет, вас будут находить. А что он не пишет, я буду находить, или мы будем находить?
Небось съебал куда-нибудь, бросив свою родню и сейчас воняет из-за границы.
Сейчас вступлю в его группу и специально эту вонь поскриншотю, чтобы оно потом не вертело жопой.
Аноним 03/04/22 Вск 16:42:44 #287 №710187 
>>710186
>Тем что он постит в телеграммчике, а не служит. Заметь как он пишет, вас будут находить. А что он не пишет, я буду находить, или мы будем находить?
А какой смысл от него будет? Он не военный, от обычного мирняка, который возьмет автомат будет скорее отрицательный профит.
>Небось съебал куда-нибудь, бросив свою родню и сейчас воняет из-за границы.
Сьебал с родней скорее всего. А что собственно он должен делать? Квартиру тоже с собой прихватить должен был?
Аноним 03/04/22 Вск 16:50:26 #288 №710189 
photo2022-03-1314-54-57.jpg
>>710187
>А какой смысл от него будет?
Это очень глупый вопрос. От каждого есть смысл, там модельеры и певцы даже вступают в строй, а он просто визжит. Ты спросил меня - в чём он не прав, я ответил.

>Сьебал с родней скорее всего.
А кто ему фото разрушенной хаты прислал? Пидорасы такие всегда бросают родню и вспоминают о ней, лишь чтобы вызвать жалость у других, чтобы набить социальных очков, а самим на неё похуй.
Аноним 03/04/22 Вск 21:13:30 #289 №710219 
16488820780742.png
В какую сторону движется солнечная система относительно ближайшей системы центавра? Расстояние растет или уменьшается? Могло ли Солнце быть рождено в системе центавра и выкинуто на ранних этапах гравитаионной пращей?
Аноним 03/04/22 Вск 21:15:05 #290 №710220 
>>710219
>В какую сторону движется солнечная система относительно ближайшей системы центавра?
Удаляется.
Аноним 04/04/22 Пнд 02:57:27 #291 №710236 
dd469ea88181708e5e680986cbe80d1c.jpg
>>710181
во всём прав
эти ебаные орки, да орки ибоармией рф и людьми их назвать уже нельзя, даже своих не похоронили за месяц стояния в гостомеле
хуй знает что они там делали, бухали, еблись в жопы или марадёрили местные ЖК, но блять факт остаётся фактом, хохлы просто в ахуе, они зашли туда и охуели лежат иваны и буряты, уже распухли, если к своим такое отношение, то что уж говорить про украинских граждан
Аноним 04/04/22 Пнд 03:11:34 #292 №710239 
6e41b06f279e0bab2e467bcde79d70c8.mp4
>>710186
>Тем что он постит в телеграммчике, а не служит.
Чел из киева писал, ему 26, пошёл записываться в тероборону, ему сказали иди нахуй сиди дома, народу и так дохуя, когда понадобишься скажут. У них 200к+ мужиков из-за бугра приехали родину защищать. То есть они живут давно в европе, польша, испания португалия, женились там и поселились, но приехали обратно, хотя могли не делать этого.
Ты представляешь себе такую ситуацию в нашей раше? Чтоб эмигранты из европы или штатов вернулись россию защищать? Да тут блять местные бегают по зауглам когда слух пошёл что мобилизация будет. Россияне патриоты только пока их жопа в тепле. Самая мерзкая хуета это кстати москвичи, самый бесполезный скам. При этом все отмазаны, ни одного из убитых или пленных вна Украине нет из москвы. Но при этом все стучат копытами по клавишам за войну.
Аноним 04/04/22 Пнд 12:10:06 #293 №710252 
>>710239
А я слышал, что там всеобщая мобилизация.
Аноним 04/04/22 Пнд 15:11:56 #294 №710256 
>>710181
>>710236
>>710239
Вырусь ебаная, пиздуйте в бэ или в по и там хрюкайте. Нехуй тут нерилейтед высирать. Искандера вам в окно.
Аноним 04/04/22 Пнд 19:57:27 #295 №710266 
1644106185621.jpg
>>709748
>Простой сценарий образование полого астероида.
>Комета сталкивался с рандомной мелкой каменюкой, после удара вся силикатная пыль медленной оседает на поверхность льда.
>Комета мигрируют во внутрь системы, где ее начинает жарить Солнце, выветривать солнечный ветер и оседать пыль.
>Пыль на границе цементируется за счет медленного испарения льда и действия солнечного ветра, формируя этакую жесткую скорлупу. Лед же в центре медленно выкипает за миллионы лет, оставляя полость.
Значит ли это, что если "подогнать" с границы солнесчной системы планету типа Ганимеда или Титана, то мы получим пустотелую скорлупу через энное время? Ебать!
То есть, шизотеория полой Луны может быть нухуя не шизо? Ебать! ЕБАТЬ!
Аноним 04/04/22 Пнд 20:07:10 #296 №710268 
>>710266
>шизотеория полой Луны может быть нухуя не шизо? Ебать! ЕБАТЬ!
У луняши масса посчитана, это камень именно такого размера что мы видим.
По идее можно ее там, хуй знает, из осмия сделать, только мы бы слышали это сейсмографами.
Аноним 05/04/22 Втр 02:03:11 #297 №710283 
675a5e8130b49b92f93264e2d1e0179e.jpg
>>710256
>пиздуйте в бэ или в по
а то что?
Аноним 05/04/22 Втр 09:12:46 #298 №710291 
crapped yourself.png
>>710256
Газоген, спок.
Аноним 05/04/22 Втр 11:17:50 #299 №710295 DELETED
>>710239
>Но при этом все стучат копытами по клавишам за войну.
Не правда твоя, мразь.
Москвичей до хуя против.
Просто их как и всех остальных в этой стране не спрашивают, когда могилизовать пошлют.
Аноним 05/04/22 Втр 11:30:06 #300 №710296 
>>710266
Нет, этот процесс актуален для маленьких тел - порядка километра в диаметре, в противном случае самогравитация обрушит все.
Аноним 05/04/22 Втр 12:35:18 #301 №710297 
>>707999 (OP)
А космонавты сосут друг у друга в космосе? И какие ощущения от эякуляции?
Аноним 05/04/22 Втр 12:49:54 #302 №710298 
>>710296
>Нет, этот процесс актуален для маленьких тел - порядка километра в диаметре, в противном случае самогравитация обрушит все
> этот процесс актуален для маленьких тел - порядка километра в диаметре
Почему 1км, а не 10 или 100? На глаз прикидываешь или есть рассчёты?
Аноним 05/04/22 Втр 12:55:53 #303 №710299 
>>710298
Порядок километра это в диапазоне 1-10 км.
Больше 10 км ледяное тело начинает деформироваться от самогравитации, а 100 км уже каменное тело начинает деформироваться. Рассчитать можно и самому прикинув среднею прочность материала и его самогравитацию.
Аноним 05/04/22 Втр 13:10:49 #304 №710300 
>>710299
>Рассчитать можно и самому прикинув среднею прочность материала и его самогравитацию.
Я не знаю формул для рассчёта
Аноним 05/04/22 Втр 13:34:54 #305 №710301 
>>710300
Допустим тело у нас сфера с полостью.
Зная плотность и размер вычисляем массу, а из нее ускорение свободного падения. Из него уже вычисляем напряжение в веществе.
Поскольку у нас материал это хлипка глина да пыль, то прочность будет советующая.
Смотрим когда напряжение будет больше прочности и находим предельный размер тела.
Аноним 05/04/22 Втр 15:50:44 #306 №710306 
>>710291
Хрюкни.
Аноним 05/04/22 Втр 18:56:21 #307 №710313 
>>710291
>пик
Это фейк?
Аноним 06/04/22 Срд 02:42:48 #308 №710335 DELETED
>>710295
>Не правда твоя, мразь.
Правда. У меня выборка широкая в скайпе в старой конфе хуева гора москваблядей и все за войну, даже блять анонимно ни одна падаль не высказалась против. Да блять чего говорить, у меня родственнички все москвабляди, все как один Zа.
Так что чеши дальше, что там кто-то против. Видели сколько против, 100 челвоек на всю москву нашлось.
Аноним 06/04/22 Срд 06:59:31 #309 №710336 DELETED
>>710335
Предлагаю сьебать на порашу. Обоим.
>там кто-то против
Я против.
Ростов на дону
Аноним 06/04/22 Срд 13:29:30 #310 №710358 
Собирались ли посылать в космос что-то на гусеницах?
Аноним 06/04/22 Срд 14:47:30 #311 №710362 
>>710358
Коротко - нет, и врятли предвидится

http://alternathistory.com/lunu-schitat-tverdoj-ili-gusenichnyj-lunohod/


Восемь автономных мотор-колес обещали несравнимо большую живучесть ходовой части. Поэтому конструкторы пересмотрели проект шасси, отказавшись от гусениц в пользу восьми ведущих колес. Пускай и мороки с ними больше, чем с двумя электродвигателями гусеничной машины, и часть драгоценного веса они отъедают, — но зато у лунохода в ходовой исчезает единая точка отказа. Подвижность сохранится при любой одиночной (и даже не одиночной) поломке ходовой. А это принципиальный момент — чинить-то некому. Но и то, — все восемь мотор-колес имели специальные пиропатроны, которые могли в случае необходимости превратить мотор-колесо в просто колесо, свободно вращающееся на оси.

Аноним 06/04/22 Срд 15:01:19 #312 №710363 
>>710362
Логично, спасибо!
Аноним 06/04/22 Срд 15:10:22 #313 №710364 DELETED
>>710335
Чмо русофобское, тебе же русским языком сказали идти отсюда на хуй в по или в бэ и там хрюкать. Здесь твое "сало уронили" нерилейтед.
Аноним 06/04/22 Срд 15:40:19 #314 №710365 DELETED
>>710364
Чего ты ожидаешь добиться вступая с ним в диалог?
Молча репорти порашу.
Аноним 06/04/22 Срд 20:34:09 #315 №710374 
Сторонники квантового бессмертия тут? Если мы не умрем, пистолет наставленный нам в голову не выстрелит и все такое, то как быть с биологическим старением? Где то в множестве вселенных будет вселенная где бессмертный гитлер до сих пор правит? Ладно, этот парень жил еще недавно, а что с Колумбом, Коперником? Где то есть мир в котором Аристотель, Платон не умерли? Но ведь никакой микробиологии не было. А что со средними веками? А что с первыми людьми разумными? Те то как выжили?
Аноним 06/04/22 Срд 20:51:46 #316 №710375 
>>710374
А вписывается ли в теорию квантового бессмертия загробная жизнь? Типа сознание не умерло а продолжает существовать каким то образом?
Аноним 07/04/22 Чтв 03:50:29 #317 №710399 
>>710375
Нет.
Сознание лишь конструкт произведенный деятельностью мясного мозга.
Твое сознание не то же что и вчера и не то же что и завтра.
"Ты" был младенцем, но это был не тот "ты" что есть сейчас. Ты не тот сейчас что был вчера, и не тот что завтра. Если ты переживешь травму мозга - "ты" ьулкгт ек иф
И кто решит кто есть ты а кто не ты?
Это шах и мат верунам в "душу".
Аноним 07/04/22 Чтв 08:08:22 #318 №710405 
image.png
image.png
Сравнимо ли управление посадочной капсулой Аполлона с вертолетом?

Бонусный вопрос: как на русском называются cyclic и collective?
Аноним 07/04/22 Чтв 08:36:21 #319 №710406 
15286555779041.jpg
23690original.jpg
>>710374
Не стоит вскрывать эту тему.

Центральный постулат квантового бессмертия: show must go on, т.е. наблюдатель продолжит наблюдать, независимо от того, поддерживается ли это текущими обстоятельствами и текущим состоянием законов физики. Природа этого наблюдателя парадоксальна. С точки зрения физической реальности его нет и не должно существовать. Он объективно не фальсифицируем, при этом мы его ощущаем, поскольку им же и являемся, и можем только придумывать словесные парадоксы про философских зомби. А с точки зрения наблюдателя иллюзорна сама реальность.

Шляпа в том, что наблюдатель на всех один. Он вообще не количественен. Это не объект, это свойство. Субъективно ты это и есть Гитлер, Аристотель и Дориан Грей, который ирл никогда не рождался. И при этом ты секунду назад и ты секунду позже - две разные сущности. Ибо времени не существует, есть лишь состояния. В том числе различные состояния памяти, локально доступной наблюдателю. Это и есть так называемая личность.

А теперь наверни математический ансамбль Тегмарка и прикинь, где там может лежать текущее состояние твоей памяти. И сколько раз оно там лежит. И при каких обстоятельствах оно там лежит. И что с тобой будет уже происходит, поскольку эти состояния, натянутые на наблюдателя, абсолютно тождественны между собой. И когда ты это поймёшь - ты охуеешь, насколько на самом деле зыбка твоя субъективная реальность, и в какие стрёмные ебеня можно улететь, если неправильно за неё цепляться. Потому что show must go on, и никуда ты с него не денешься.

Что почувствует кот Шрёдингера, если в лаборатории взорвётся атомная бомба? И куда он после этого вылезет?

Но ты об этом медитируй поменьше и помедленнее. Тибетские школы такие гротескно-нелепые не из-за глупости, а чтобы ты раньше времени не ёбнулся. Нас спасает суетность и лень.
Аноним 07/04/22 Чтв 08:41:24 #320 №710407 
>>710406
Я пожалел о времени потраченном на прочтении этого поста.
Если кто увидит мой пост прежде - считайте это предупреждением.
Аноним 07/04/22 Чтв 08:41:51 #321 №710409 
>>710407
*на прочтение
быстрофикс
Аноним 07/04/22 Чтв 10:22:40 #322 №710415 
>>710362
Ну ещё и проходимость у 8 колёс больше по сухому грунту и сложному рельефу, чем у гусениц.
Аноним 07/04/22 Чтв 10:45:24 #323 №710416 
image.png
>>710415
Момент - у восьми колес с независимой подвеской. Это важно.
Я собирал машину из 10 колес без подвески, она ничего не может преодолеть.
Пик рилейтед.
Аноним 07/04/22 Чтв 11:22:18 #324 №710417 
>>710405
>cyclic
По моему так и называют - рукоятка управления
>collective
Шаг-газ или типа того.
Аноним 07/04/22 Чтв 11:35:52 #325 №710419 
>>710416
А что сложного в подвеске? Единственное проблема она должна выдерживать криогенные температуры без потери своих качеств. Проблема решена давно и все упирается в цену.
Другое дело на каждое колесо нужен свой привод(считай двигатель) и сложная система управления для синергии колес. Вот и тут ждут когда допилят ИИ.
Аноним 07/04/22 Чтв 11:47:31 #326 №710421 
>>710405
>Сравнимо ли управление посадочной капсулой Аполлона с вертолетом?
Нет.
Управление вертолетом идет через гироскопические эффекты, геометрия винта в целом + тяга двигателя. Это порождает особую систему степеней свободы, где вращательные и поступательные движения сложно взаимосвязаны.
У посадочных модулей простое управление без этой хууйни и там отдельно на поступательное и вращательное движение (из-за особенностей ориентации управление вращением идет по четырем параметрам.).
Аноним 07/04/22 Чтв 16:34:55 #327 №710439 
>>710416>>710419
У планетоходов всегда независимая подвеска. Причём её можно сделать и без пружин, на одних рычагах, чтобы просто каждая ось или пара колёс свободно болталась и вставала бы как ей удобнее.
Аноним 07/04/22 Чтв 18:16:42 #328 №710450 
>>710419
>и сложная система управления для синергии колес. Вот и тут ждут когда допилят ИИ.
Да ты же имбецил.
Аноним 07/04/22 Чтв 20:10:17 #329 №710454 
>>710417
>>710405
>cyclic
ручка продольно-поперечного управления
> collective
рычаг общего шага автомата перекоса
Представьте себе.
Аноним 07/04/22 Чтв 23:05:00 #330 №710461 
>>710374
Это все пустые маняфантазии, нет смысла разбираться в бреде сумасшедшего.
Аноним 08/04/22 Птн 02:44:40 #331 №710469 
оче смешно 11.png
>>710399
>Если ты переживешь травму мозга - "ты" ьулкгт ек иф
цук, ору
Аноним 08/04/22 Птн 13:39:19 #332 №710499 
image.png
image.png
>>707999 (OP)
Почему поверхность Европы имеет красноватый оттенок, а Энцелад, тоже с ледяной коркой над океаном, практически весь белый?
Аноним 08/04/22 Птн 15:35:58 #333 №710507 
>>710499
На Европу Ио срет своим серным говном.
Аноним 08/04/22 Птн 16:23:53 #334 №710512 
>>709791
>уничтожает другие цивилизации
Хуй знает что там в сериале, но в книжке они помогали эволюционировать, а не уничтожали.
Аноним 08/04/22 Птн 20:53:42 #335 №710541 
Почему во всем мире отказались от ядерной энергетики? Да, я понимаю, что пара обсеров было, которые загадили некоторую территорию. Но почему надо отказываться? Может лучше учесть ошибки и сделать нормально?
Аноним 08/04/22 Птн 21:44:00 #336 №710546 
>>710541
Ядерные технологии закрытые и сильно не рыночные, по факту их контролируют монополии. В добавок благодаря совку и пиндосам она обзавелась не очень хорошими традициями и методами взаимодействия с другими отраслями. Короче слишком высокие издержки на договоры.
Аноним 08/04/22 Птн 21:58:33 #337 №710548 
Почему Вселенная не образовалась сразу с тепловым равновесием у себя?
Аноним 08/04/22 Птн 22:10:38 #338 №710549 
>>710541
Толерантные еврозеленые как раз и напирают на эти обсеры, еще во времена Чернобыля поднялся визг, что после аварии на ЧАЭС вся Европа станет необитаемой. К тому же еврозеленым в верхах тоже хочется денег и активно пиарят "зеленую" энергетику для соответствующих попилов.
Аноним 09/04/22 Суб 00:36:46 #339 №710553 
>>710541
>Почему во всем мире отказались от ядерной энергетики?
Не во всем мире, а только в толерастической западной европке, да и то не везде. В той же Франции весьма развитая атомная энергетика. Ну а отказываются, потому что гнилой евроистеблишмент хочет пилить бабло на "зеленой энергии".
Аноним 09/04/22 Суб 01:54:39 #340 №710554 
>>710548
Хуя у тебя вопросы.
Аноним 09/04/22 Суб 23:14:23 #341 №710626 
image.png
>>710454
Жаль, что cyclic не переводится как суслик.
Аноним 10/04/22 Вск 01:00:01 #342 №710635 
15408312893970.jpg
>>710499
лёд здорового человека и лёд курильщика
Аноним 10/04/22 Вск 03:29:13 #343 №710643 
>>710507
Там кстати на орбите пиздец воняет этой сернистой гавной.
Ио главная вонючка СС. А на Европе живут газонюхи получается.
Аноним 10/04/22 Вск 04:20:13 #344 №710644 
>>710643
Не "на Европе", а "в Европе", грамотей.
Извините.
Аноним 10/04/22 Вск 04:42:27 #345 №710645 
>>710644
Орнул сукаааа!!!
Аноним 10/04/22 Вск 15:23:17 #346 №710698 
1. Обратно приземляющиеся ракеты Маска - сфера применимости?
2. Есть разработки ядерных движков, каким образом на них бы происходил взлёт? Там же наверное не может быть такой тяги чтобы вертикально взлетать. Как самолёт?
Аноним 10/04/22 Вск 15:40:27 #347 №710699 
>>710698
>1. Обратно приземляющиеся ракеты Маска - сфера применимости?
Э-э-э... Да?
>2. Есть разработки ядерных движков, каким образом на них бы происходил взлёт? Там же наверное не может быть такой тяги чтобы вертикально взлетать. Как самолёт?
Буквально ядерные взрывы.
https://en.wikipedia.org/wiki/Nuclear_pulse_propulsion
https://ru.wikipedia.org/wiki/Орион_(МКА)
Аноним 10/04/22 Вск 16:30:04 #348 №710703 
>>710698
1. PR
2. Ну почему же, были проекты маршевых ЯРД сврехвысокой тяги. Там проблема не в тяге, а в надежности, нужна устойчивости активной зоны к высоким температурам и давлениям и как конструктивно разделить выхлоп и радиоактивный контур. Если не отделять радиоактивность от выхлопа, то вообще никаких конструктивных проблем достичь сверхвысокой тяги нет.
Аноним 10/04/22 Вск 19:49:20 #349 №710717 
Elon Musk MBTI.jpg
Была новость про изобретение керамики, которая может регенерировать/само исцеляться.

В чем суть? Сфера применения?

Аноним 10/04/22 Вск 20:46:46 #350 №710719 
>>710698
>2. Есть разработки ядерных движков, каким образом на них бы происходил взлёт? Там же наверное не может быть такой тяги чтобы вертикально взлетать. Как самолёт?
Очевидные ядерные прямоточки, использующие рабочее тело из атмосферы. Нет проблем гиперзвукового горения, есть проблемы температурных ограничений и ядерного выхлопа.
Аноним 10/04/22 Вск 22:12:38 #351 №710729 
>>710717
>регенерировать/само исцеляться
Это слишком громко сказано.
Суть новости в том, что у некоторых перовскитных структур дефекты самоуничтожаются на поверхности границы. Такое бывает у обычных структур, но в этом случае требуется особые условия, отчего дефекты копятся, пока не придут в термодинамическое равновесие или дефекты станут критичными и разрушат структуру. Это является главным фактором старения и усталости материалов. У структур из новости дефекты уничтожаются еще до того как они придут в термодинамическое равновесие, что позволяет структуре как бы "омолаживаться".

Короче, такая керамика не будет терять своих свойств со времен и даже держать определенный стресс. Во влажных фантазиях тут профит для полупроводников, поскольку их деградация со временем развивается драматически. Ну и второе это различные защитные покрытия, которые подвержены различным стрессам, в первую очередь радиационным и ударным.
Аноним 10/04/22 Вск 22:31:37 #352 №710730 
>>710729
Разве керамика имеет потенциал для полупроводников?
Аноним 10/04/22 Вск 23:04:21 #353 №710732 
>>710730
Многие керамики внезапно полупроводники. Да и всякие полупроводниковые приборы требует вспомогательных вещей, вроде банальной подложки и изолятора.
Аноним 10/04/22 Вск 23:33:53 #354 №710736 
1359325862119.png
>>710717
>регенерировать/само исцеляться.
>керамика
Был слух, что тебя драли десять негров. Нет никаких оснований предполагать, что это пиздёж.
Аноним 11/04/22 Пнд 01:21:51 #355 №710740 
image.png
Почему нет переходов сразу в плазму из чего угодно кроме газа? Разве нельзя сильно зарядить, ионизировать твердое тело чтобы сразу ионы вылетали? Как сублимация, но с еще большей энергетикой?
Скорей всего ответ простой и вопрос этот туповат, но я пока не понимаю.
Аноним 11/04/22 Пнд 02:47:59 #356 №710741 
>>710740
Потому что табличку делал мудак, а переходы такие есть.
Аноним 11/04/22 Пнд 05:40:11 #357 №710744 
>>710499
Инцелад распидорашивается и покрывается льдом оседающим из своих же фонтанчиков, а Цеевропа имеет слишком толстую корку
Аноним 11/04/22 Пнд 21:39:30 #358 №710782 
>>710740
>Разве нельзя сильно зарядить, ионизировать твердое тело чтобы сразу ионы вылетали?
Есть. При выполнении пятилетки по выработке энергии за 3 секунды. Ядерный взрыв называется
Аноним 12/04/22 Втр 12:13:15 #359 №710809 
>>710782
Тащемта есть менее экстранормальные вещи как ионное распыление, которое уже давно используют в народном хозяйстве.
Аноним 12/04/22 Втр 12:36:39 #360 №710812 
Поясните про квантовость и наблюдателя.
Что за наблюдатель, и как без него одновременно все результаты бывают?
По-двачерски, пожалуйста.
Аноним 12/04/22 Втр 12:54:29 #361 №710817 
>>710812
Наблюдатель - это любая частица взаимодействующая с наблюдаемым обьектом.
>По-двачерски, пожалуйста.
Эм... И она может быть даже из говна.
Аноним 12/04/22 Втр 13:25:19 #362 №710822 
>>710817
Нет, просто не понимаю, как без "наблюдателя" получается интерфер... как там, решетка, а с наблюдателем нет?
Аноним 12/04/22 Втр 13:30:06 #363 №710824 
>>710822
Читай про статистическую интерпретацию волновой функции.
https://ru.wikipedia.org/wiki/%D0%A1%D1%82%D0%B0%D1%82%D0%B8%D1%81%D1%82%D0%B8%D1%87%D0%B5%D1%81%D0%BA%D0%B0%D1%8F_%D0%B8%D0%BD%D1%82%D0%B5%D1%80%D0%BF%D1%80%D0%B5%D1%82%D0%B0%D1%86%D0%B8%D1%8F_%D0%B2%D0%BE%D0%BB%D0%BD%D0%BE%D0%B2%D0%BE%D0%B9_%D1%84%D1%83%D0%BD%D0%BA%D1%86%D0%B8%D0%B8
Аноним 12/04/22 Втр 13:34:10 #364 №710825 
>>710824
Пиздец.
Надо подумать об этом.
Двачерские интерпретации сложно будет придумать, но я понимаю теперь почему.

Давай левый вопрос - почему у тебя такая некрасивая ссылка? Я же видел что можно сделать нормально.
Аноним 12/04/22 Втр 13:36:11 #365 №710826 
>>710825
>Давай левый вопрос - почему у тебя такая некрасивая ссылка? Я же видел что можно сделать нормально.
Потому что мне это нахуй не нужно. Скопировал из адресной строки как есть. Зачем мне заморачиваться, делая ее "красивой"?
Аноним 12/04/22 Втр 13:43:12 #366 №710831 
>>710826
>Потому что мне это нахуй не нужно. Скопировал из адресной строки как есть. Зачем мне заморачиваться, делая ее "красивой"?
Так тебе нахуй не нужно и отвечать кому-то. Ты же не для себя это делаешь. Странно просто.
Не переживай, я не пытаюсь наезжать, просто спросил. Спасибо за ответ.
Аноним 12/04/22 Втр 14:08:06 #367 №710834 
Насколько изменится температура на земле если все под поверхностью коры вдруг станет комнатной температуры?
Аноним 12/04/22 Втр 14:08:30 #368 №710835 
>>710812
Дауны не способны понять как работает мир, а потому для объяснения используют магию наблюдателя, который в свою очередь фактом наблюдения магически влияет на наблюдаемое событие.

Если проще - есть иная причина - дуализма частиц не существует.
Аноним 12/04/22 Втр 14:27:06 #369 №710838 
>>710835
Ты не ответил на то почему при "наблюдении" меняется результат и какое оно это наблюдение.
Ты хотел выглядеть как кто-то кто несет откровение, но показал пустословие.
Аноним 12/04/22 Втр 14:50:30 #370 №710842 
>>710838
>при "наблюдении" меняется результат
Он не меняется, шизик.
Аноним 12/04/22 Втр 14:54:08 #371 №710843 
>>710838
Наблюдение это взаимодействие. Приложив линейку к пинусу ты чучть-чуть сдвинешь его в сторону. А чтобы наблюдать какую-нибудь частицу к ней нужно приложить другую частицу, вот результат и меняется.
Аноним 12/04/22 Втр 15:24:00 #372 №710844 
>>710842
Меняется, это доказано уже.
От тебя, именно ты тут шизик, ответ уже не интересен.
Аноним 12/04/22 Втр 15:24:24 #373 №710845 
>>710843
А как наблюдение-то производится?
Аноним 12/04/22 Втр 15:43:58 #374 №710846 
16490180033090.png
>>710838
По Божьей воле матрица нашей вселенной построена так, чтобы не перегружать подругзку текстур в макро и микромасштабах, поэтому когда венец творения - человек; воля, облочённая во плоти и управляемая разумом; душа - наблюдает за чем-то конкретным, вселенная подстраивается под этого наблюдателя и включает прорисовку текстур получше. Но как только носитель божьей искры перестаёт наблюдать, подгрузка текстур отключается и шейдеры, вычисления, графоны и т.д. нашего пространства переходят в режим упрощённый, оптимизированный под быстродействие, нежели чем под красоту.
Аноним 12/04/22 Втр 16:07:51 #375 №710850 
Как и где образуются лучи при съёмке или наблюдением невооружённым взглядом?
Аноним 12/04/22 Втр 16:08:20 #376 №710851 
undefined
ОДИН СПУТНИК МАРСА УЖЕ НА НЕГО УПАЛ И СКОРО УПАДЁТ ФОБОС
@
ПОТОМУ ЧТО ТАК РАБОТАЕТ ОРБИТАЛЬНАЯ МЕХАНИКА И ГРАВИТАЦИЯ

ЛУНА ОТДАЛЯЕТСЯ ОТ ЗЕМЛИ И В БУДУЩЕМ ОТОРВЁТСЯ ОТ НЕЁ И УЛЕТИТ
@
ПОТОМУ ЧТО ТАК РАБОТАЕТ ОРБИТАЛЬНАЯ МЕХАНИКА И ГРАВИТАЦИЯ


Так, абажжи...
Аноним 12/04/22 Втр 16:22:32 #377 №710853 
>>710851
Просто ты бинарный долбоеб и тебе недоступно понимание законов небесной механики
Аноним 12/04/22 Втр 16:26:59 #378 №710855 
>>710845
Сталкивают например.
Аноним 12/04/22 Втр 16:49:01 #379 №710856 
>>710851
ГРАВИТАЦИОННОЕ ВЗАИМОДЕЙСТВИЕ СЛИШКОМ СИЛЬНОЕ
@
ВС
@
ГРАВИТАЦИОННОЕ ВЗАИМОДЕЙСТВИЕ СЛИШКОМ СЛАБОЕ
@
СЛОЖНА РЯЯ
Аноним 12/04/22 Втр 16:49:09 #380 №710857 
>>710831
>Так тебе нахуй не нужно и отвечать кому-то.
Нужно, раз отвечаю. Это должно быть очевидно. Если бы мне было не нужно или, правильнее сказать, если бы я был в этом совсем не заинтересован, я бы никогда не заходил на харкач и ничего бы не постил. Эта предельно простая логическая связь была бы тобой воспринята, если бы, перед тем как задавать тупой вопрос, ты хотя бы минимально напряг свои извилины.
>Ты же не для себя это делаешь.
Для себя. Для кого же еще, кроме как для себя, мне это делать? Или ты наивно полагаешь, что меня побуждает что-то писать в ответ на твои не очень умные вопросы искреннее желание поделиться с тобой какими-то знаниями или в чем-то помочь? Конечно нет.
>Странно просто.
Странный ты, доебавшийся до вида ссылки.
Аноним 12/04/22 Втр 20:57:01 #381 №710936 
>>710851
>ЛУНА ОТДАЛЯЕТСЯ ОТ ЗЕМЛИ И В БУДУЩЕМ ОТОРВЁТСЯ ОТ НЕЁ И УЛЕТИТ
А я наоборот слышал, что отдаления прекратится и Луна приземлится обратно.
Аноним 12/04/22 Втр 23:30:28 #382 №710946 
>>710851
ПРЕДЕЛ РОШАНА
@
НЕ РАЗРЫВАЕТ ЛУНУ МАРСА СПОКОЙНО НА НЕГО УПАВШУЮ
Аноним 13/04/22 Срд 08:22:38 #383 №710965 
>>710946
Какого еще Равшана???
Аноним 13/04/22 Срд 08:43:09 #384 №710966 
>>710851
>ОДИН СПУТНИК МАРСА УЖЕ НА НЕГО УПАЛ
Какой, когда?
>СКОРО УПАДЁТ ФОБОС
>@
>ПОТОМУ ЧТО ТАК РАБОТАЕТ ОРБИТАЛЬНАЯ МЕХАНИКА И ГРАВИТАЦИЯ
Да.

>ЛУНА ОТДАЛЯЕТСЯ ОТ ЗЕМЛИ И В БУДУЩЕМ ОТОРВЁТСЯ ОТ НЕЁ И УЛЕТИТ
Нет, не улетит. Отдаление замедляется с расстоянием потому что отдаление обусловлено приливами.

>ПОТОМУ ЧТО ТАК РАБОТАЕТ ОРБИТАЛЬНАЯ МЕХАНИКА И ГРАВИТАЦИЯ
Именно так.
Аноним 13/04/22 Срд 08:44:56 #385 №710967 
>>710857
>доебавшийся
Какого хуя ты это доёбом считаешь?
Только быдло воспринимает критику в штыки.
Аноним 13/04/22 Срд 10:38:15 #386 №710968 
>>710967
>Какого хуя ты это доёбом считаешь?
Потому что это доеб.
>Только быдло воспринимает критику в штыки.
Покажи всем еще, какое ты небыдло. Тебе ведь так важны эти ярлычки, да, овощ?
Аноним 13/04/22 Срд 11:05:27 #387 №710969 
>>710850
Ты про радиальные лучи от ярких источников?
Так это дифракция света.
В случае объективов это граница диаграммы у линзы или крепления зеркала у телескопов.
В случае глаза человека это ресницы и внезапно роговица.
Аноним 13/04/22 Срд 14:36:02 #388 №710977 
>>710968
>Потому что это доеб.
Нет, не доёб. Это у тебя доёб ко мне за критику.
>Покажи всем еще, какое ты небыдло. Тебе ведь так важны эти ярлычки, да, овощ?
Вот ты и попалось, быдло ебаное. Ты не смог скрываться так долго.
Скорее бы вас в рабство загнали, скоты ебаные, вы не должны иметь права слова.
Аноним 13/04/22 Срд 15:16:56 #389 №710980 
>>710936
И засчёт чего Луна начнёт приближаться к Земле? Там колебания что ли?
Аноним 13/04/22 Срд 15:18:03 #390 №710981 
>>710977
Чмо тупое, успокойся.
Аноним 13/04/22 Срд 16:26:18 #391 №710984 
>>710977
>Нет, не доёб.
Нет, это доеб.
>Вот ты и попалось, быдло ебаное.
Ты очень туп и не умеешь в тралинг. Попробуй еще раз, глупенький огуречик.
Аноним 13/04/22 Срд 21:57:24 #392 №710993 
>>710981
Чмо тут ты, быдло ебаное. Ты объективно являешься биомусором, ты не то что бесполезен для человечества, ты объективно вреден. Паразит ебаный. Чтоб ты сдох.

>>710984
Ты тоже. Как меня заебало засилье быдла кругом, жалко, что ковид не выкосил вас, ублюдков.
Аноним 14/04/22 Чтв 06:02:00 #393 №711012 
image.png
Металлургический вопрос. Есть ли какие-нибудь йоба-сплавы или еще какая хитрая хрень которую можно только в невесомости запиливать?
Аноним 14/04/22 Чтв 09:14:29 #394 №711035 
>>710993
Пробуй тоньше.
Аноним 14/04/22 Чтв 11:50:26 #395 №711043 
>>711012
Чтоб прям только в невесомости вряд ли. Разве что кристаллы выращивать размером с дом.
Все профиты микрогравитации упирается в размеры, в ней можно клепать огромные конструкции.
Аноним 14/04/22 Чтв 15:51:09 #396 №711054 
>>710980
За счет излучения гравитационный волн.
Аноним 14/04/22 Чтв 16:01:18 #397 №711055 
Есть перекись водорода.
А бывает ли НЕДОКИСЬ водорода?
Аноним 14/04/22 Чтв 16:19:06 #398 №711059 
>>711055
Нет, но им можно считать гидроксил радикал.
Аноним 14/04/22 Чтв 22:15:20 #399 №711074 
>>710993
Проигрунькал.
>>711054
А оно разве с увеличением расстояния усиливается, а не ослабевает?
Аноним 14/04/22 Чтв 23:29:44 #400 №711077 
>>711074
Замедления через излучение убывает медленнее, чем приливное ускорение.
Аноним 15/04/22 Птн 04:37:50 #401 №711081 
DjAjYtpRa10.jpg
>>711043
>Разве что кристаллы выращивать размером с дом
мимо другой анон
Вопрос в догонку, есть ли какие-то профиты от кристаллов размером с дом?
Аноним 15/04/22 Птн 06:44:28 #402 №711082 
image.png
image.png
image.png
image.png
>>711081
По идее для электроники профитно, у тебя чем больше блинчик кремния, тем меньше незадействованной площади на нем.
Аноним 15/04/22 Птн 10:15:43 #403 №711088 
>>711081
При выращенном монокристалле с правильной структурой кристаллической решётки, с поляризацией и всем прочим, при огромных размерах, можно получить сверхпроводник, который ещё может быть и сверхпрочным. А при некоторых условиях, теоретически, действительно большие такие вот монокристаллы могут защитить от радиации
Аноним 15/04/22 Птн 10:43:43 #404 №711090 
>>711088
Так вроде наоборот монокристаллы говно, для железа так точно, потому сплавы и рулят, что чистый элемент хуйня.
Аноним 15/04/22 Птн 11:16:41 #405 №711091 
>>711077
Откуда инфа?
Аноним 15/04/22 Птн 11:41:23 #406 №711093 
>>711090
Все зависит какие кристаллы.
Для пьезофигни нужны монокристаллы.
Для полупроводников тоже.
Все профиты графена актуальны только для цельного листа и следовательно для монокристалла.
Профиты нанотрубок так же.
Даже ебучий полиэтилен будет убер веществом, если он будет состоять из одной молекулы. И так со всеми упорядоченными полимерами.
Аноним 15/04/22 Птн 11:56:28 #407 №711095 
>>711093
Тут спорить не стану.
Аноним 15/04/22 Птн 12:19:10 #408 №711097 
>>711093
Уже почти есть же!
>Сшитый полиэтилен
Аноним 15/04/22 Птн 17:40:00 #409 №711123 
Откуда на Земле столько, блядь, воды? На остальных планетах её же хуй да нихуя. На Европе разве что есть. Или я не прав?
Аноним 15/04/22 Птн 17:42:44 #410 №711124 
>>711123
Не сдуло.
Аноним 16/04/22 Суб 02:25:37 #411 №711165 
два чая.jpg
>>711093
Добра тебе анон. Разложил по полочкам.
Аноним 16/04/22 Суб 02:32:11 #412 №711166 
earth2-750x991.jpg
>>711123
>Откуда на Земле столько, блядь, воды?
На венере есть, только она вся выкипела и в атмосфере.
Да дохуя где есть больше чем на земляшке. На вот, изучай, раз в гугл не можешь.
Аноним 16/04/22 Суб 05:55:33 #413 №711167 DELETED
анoны, пoжалyйста, пoмoгитe
я видeл, чтo тyт oдин yжe пoпpoшайничал, так чтo пoймy, eсли наxyй пoшлётe..
y мeня мoй кoтoфeй забoлeл. зoвyт Гoша - саpкoма жeлyдка, тpи нeдeли назад oбнаpyжили..
сам стyдeнт, eщё нe pабoтаю, пoчти ниxyя нe имeю, а тo, чтo имeл, yжe пpoдал - смаpтфoн, нeкoтopyю oдeждy, кoмплeктyющиe oт пк, кoтopыe xoть чeгo-тo стoили. так жe влeз в дoлги. пытался в микpoзаймы или кpeдиты - ниxpeна нe дают из-за oтсyтствия pабoты. poдитeлeй нeт, тoлькo бабyшка, кoтopая пoчти всю пeнсию на сeбя тpатит - лeкаpства, eда, кoммyналка. в oбщeм, нe пoмoщница..
так наскpёб 16 тысяч. на oпepацию нyжнo 18, eщё сpаныx 2-x кoсаpeй нe xватаeт. а oпepация на сeгoдня (на 11 дня) назначeна бляять...

yмoляю, анoны, xoтя бы pyблём пoмoгитe пoжалyйста.

5521.4003.7923.2235

я дoгадываюсь, чтo нeкoтopыe пpeдлoжат забить на кoтoфeя - нe ваpиант. Гoша сo мнoй yжe пoлжизни, этo самoe дopoгoe сyщeствo для мeня. и сeйчас, смoтpя, как oн мyчаeтся, xoчeтся наxyй вскpыться...

мeня скopee всeгo сeйчас забанят к xepам, пoэтoмy извинитe, eсли oтвeтить нe смoгy..

Аноним 16/04/22 Суб 08:28:36 #414 №711177 
https://m.hightech.plus/2022/04/06/pentagon-o-kontaktah-s-nlo-zafiksirovani-radiacionnie-ozhogi-i-sluchai-neobyasnimoi-beremennosti
Как-то обошли вниманием посреди информационного шума последних дней, даже в сначевском уфотреде ничего про это не сказали. Кто как думает, что все это может значить?
Аноним 16/04/22 Суб 08:50:02 #415 №711178 
>>711177
Это значит, что-то кто-то не принял таблетки.
Аноним 16/04/22 Суб 08:54:08 #416 №711181 
>>711177
А ничего не значит. Чего ты от внушаемых вояк хочешь, у которых суеверия на суевериях?
Аноним 16/04/22 Суб 10:48:24 #417 №711188 
>>711178
Сложно списать 1500-страничный доклад, сделанный по рассекреченной программе исследования НЛО Минобороны США, на "непринятие таблеток".
>>711181
То же самое. Какие суеверия могут приводить к написанию 1500-страничного доклада на основе работы рассекреченной программы Минобороны США? Выглядит все пиздец как серьезно, и на этот раз сложно все это списать на "таблетки" или "суеверия". По суевериям не создают засекреченные программы исследования, это даже звучит слишком бредово и неправдоподобно.
Аноним 16/04/22 Суб 11:20:42 #418 №711190 
>>711188
Ну доклад и доклад. Понаписали что полтора пилота фуфайтеров насмотрелись, повыдумывали причин и все. Реальной работы там нет.
Если бы что-то на самом деле было, моченые бы уже давно детектили по всему миру и публиковались гипотезы по феноменам.
Да и с распространением мобилок среди масс сейчас бы кишмя кишели интернеты пруфами, а количество наоборот ниспадает, т.к. фейки/артефакты легко палятся.
Аноним 16/04/22 Суб 16:54:06 #419 №711206 
>>711190
>Понаписали что полтора пилота фуфайтеров насмотрелись
Там не про пилотов, а про обычных американцев, сталкивавшихся с НЛО и подвергшихся в результате этого контакта различным биологическим эффектам.
>повыдумывали причин
Ну официальным властям, особенно минобороны, наоборот выгоднее выдумать чему-либо необъяснимому какую-нибудь слишком банальную причину, чтобы отмахнуться от назойливой общественности и не привлекать внимания к тому, к чему они не хотят, чтобы привлекалось общественное внимание (по каким угодно причинам). К тому же, в докладе перечислены факты, т.е. конкретные случаи, а не просто спекуляции на тему.
В общем, все это весьма странно и непонятно. Я для себя пока еще не нашел адекватного объяснения всей этой хуйне. Версия с зелеными человечками звучит слишком дико, но и ее нельзя огульно отвергать, тем более при таких обстоятельствах.
Аноним 16/04/22 Суб 17:09:06 #420 №711210 
>>711206
Самое забавное, что ты не задумываешься о том, что как раз они могут запросто насрать в уши чтобы был повод людям про химтрейлы и прочие теории заговора трещать.
Моченые почему-то не публикуют ничего на эту тему. Воены могут хоть "будда в Пекине снес золотое яйцо" с пруфами написать.
Аноним 16/04/22 Суб 17:37:48 #421 №711215 
>>711210
>что как раз они могут запросто насрать в уши чтобы был повод людям про химтрейлы и прочие теории заговора трещать
Это тоже имеет место, да, но тут не просто кто-то что-то спизданул - тут целый доклад на 1500 страниц с материалом, взятым из рассекреченной программы минобороны. Это немножко другой уровень.
>Моченые почему-то не публикуют ничего на эту тему.
Так в Минобороны США 100% работают ученые, тем более они принимают участие в таких программах, как эта по исследованию НЛО. Изучением перечисленных в докладе случаев, как и составлением самого доклада наверняка, занимались как раз ученые, потому что здесь именно их сфера. Вояки в биологических эффектах мало что понимают. Поэтому в данной истории моченые определенно отметились. А во всяких "высокоимпактных" нэйче и саенсах публиковать такое не с руки, "навучное сообщество" может не понять и засмеять (современное западное "навучное сообщество" открыто ведь для рассмотрения любых фактов и аргументов, да-да, особенно тех, которые не вписываются в прогрессивную повесточку).
Аноним 16/04/22 Суб 17:49:11 #422 №711219 
>>711215
Ладно, полистаю, ты прав, не надо все так явно отрицать.
Аноним 17/04/22 Вск 02:19:40 #423 №711288 
>>711215
>Это тоже имеет место, да, но тут не просто кто-то что-то спизданул - тут целый доклад на 1500 страниц с материалом, взятым из рассекреченной программы минобороны.
От этого шизофантазии сбежавших из дурки не становятся меньшими шизофантазиями сбежавших из дурки. Вот из-за такой хуйни лоботомия была в пиндостане довольно популярная.
Аноним 17/04/22 Вск 08:56:08 #424 №711295 
>>711288
>От этого шизофантазии сбежавших из дурки не становятся меньшими шизофантазиями
Все что не вписывается в твой уютный манямирок можно слепо обозвать "шизофантазиями" и продолжать жить с прежними установками блаженного овоща в голове, я знаю. Так гораздо легче, чем напрягать мозг и пытаться анализировать информацию. Но лично мне интересно узнать, как оно на самом деле, по-настоящему разобраться в ситуации. Тебе, очевидно, ближе первый вариант, поэтому я вообще не вижу смысла в твоей реакции на мой пост. Это же всего лишь "шизофантазии", зачем на них обращать внимание, тратить свое время на их обсуждение? Лучше иди в дотку поиграй.
Аноним 17/04/22 Вск 09:04:30 #425 №711296 
А что случилось с https://what-if.xkcd.com
Почему он перестал постить? Умер что ли?
Аноним 17/04/22 Вск 09:51:38 #426 №711305 DELETED
>>711295
Вероблядь, не трясись так.
Аноним 17/04/22 Вск 10:15:29 #427 №711317 DELETED
Шизик вернулся, репортьте его "Шитпостинг" >>711305
Не нужны такие на доске, ничего не добавляет к дискуссии, просто агрессивно огрызается.
Аноним 17/04/22 Вск 11:45:26 #428 №711324 DELETED
>>711305
Я не хотел тебя обоссывать, просто написал как есть. Извини, если тебя это так сильно обидело.
>>711317
Не триггерись так сильно, глубоко вдохни, переведи дух. Неужели твой манямирок настолько хрупкий, что при малейшем намеке на другое мнение ты начинаешь ронять кал и громко кричать, требуя забанить всех неугодных и создать тебе сейфспейс?
Аноним 17/04/22 Вск 12:16:42 #429 №711325 DELETED
>>711324
Ты не мнение высказываешь, а шитпостишь. Этот пост >>711305 ничем не отличается от
>пук
Никакого конструктива, никакой дискуссии. Это шитпостинг в чистом виде. Если можешь что-то по делу сказать - говори. Не можешь - в /b/ какай.
Аноним 17/04/22 Вск 12:24:56 #430 №711326 DELETED
>>711325
Я анон, который написал этот пост >>711295, если что. Мы с тобой просто не поняли друг друга. Ну, вернее, я не понял, думал, ты быканул на меня и поддакнул тому дегенерату. Оказалось, что это не так.
Аноним 17/04/22 Вск 18:26:51 #431 №711355 
Можете объяснить, почему началось расширение?
Аноним 17/04/22 Вск 18:34:51 #432 №711356 
>>710406
> А теперь наверни математический ансамбль Тегмарка и прикинь, где там может лежать текущее состояние твоей памяти. И сколько раз оно там лежит. И при каких обстоятельствах оно там лежит. И что с тобой будет уже происходит, поскольку эти состояния, натянутые на наблюдателя, абсолютно тождественны между собой. И когда ты это поймёшь - ты охуеешь, насколько на самом деле зыбка твоя субъективная реальность, и в какие стрёмные ебеня можно улететь, если неправильно за неё цепляться. Потому что show must go on, и никуда ты с него не денешься.
Ниче не понял. Может кто-то объяснить что он имеет в виду?
Аноним 17/04/22 Вск 18:53:26 #433 №711360 
>>711190
> Да и с распространением мобилок среди масс сейчас бы кишмя кишели интернеты пруфами
> ряяяя врете
Жесть, ну ты и верун. Попробуй на свой телефончик сфотокать луну в полнолуние, увидишь свои "пруфы"
Аноним 17/04/22 Вск 18:59:05 #434 №711362 
>>711215
>>711206
Всё объясняется куда проще, научное сообщество это сборище политизированных, ангажированных авторитетов (жрецов), гендерных учёных, популяризаторов(современных попов) и прочих сойджеков, естессно никто из них не будет заниматься темой НЛО. Вон почитай про двигатель который НИМОЖЕТ РАБОТАТЬ ФИЗИЧЕСКИ РЯЯЯЯЯСКОЗАЛ, там такие отрицалы сидят шо пздц и грантопилы
Аноним 17/04/22 Вск 19:01:03 #435 №711364 DELETED
>>711305
> Вероблядь, не трясись так.
Соевая хуета порвалась
Аноним 17/04/22 Вск 21:51:11 #436 №711375 
вопрос назрел - почему человечество не тратит ресурсы и силы на терраформирование Нептуна? Только потому что далеко лететь? Или есть какие то объективные причины?
Аноним 17/04/22 Вск 23:57:52 #437 №711382 
>>707999 (OP)
Разлет вещества от сверхновой происходит на релятивистских скоростях или медленно?
Аноним 17/04/22 Вск 23:59:22 #438 №711383 
>>711375
Да, далеко. Температуры низкие. Состав атмосферы сложноватый.
Ну и ТО ЧТО ЭТО ЕБАНЫЙ ГАЗОВЫЙ ГИГАНТ тоже не сильно помогает.
Аноним 18/04/22 Пнд 00:01:43 #439 №711384 
Почему в космос летаем на ракетах? Почему нельзя на самолёте?
Аноним 18/04/22 Пнд 01:32:54 #440 №711388 
>>711384
Можно.
Аноним 18/04/22 Пнд 04:51:23 #441 №711390 
15374460834340.jpg
>>711384
>Почему нельзя на самолёте?
Понятно конечно что тред тупых вопросов, но не на столько же.
Это уже не смешно, тупо кринж и всё.
Аноним 18/04/22 Пнд 04:54:38 #442 №711391 
>>711382
Последние данные были по какой-то сверхновой, что до 0,5 скорости света.
А так по разному. Наверно там много факторов.
Аноним 18/04/22 Пнд 04:55:46 #443 №711392 
>>711355
Захотелось ему так, вот и началось.
Аноним 18/04/22 Пнд 08:52:32 #444 №711419 
Можно ли было спасти экипажи Челленджера и Колумбии?
Про Волкова, Пацаева, Добровольского и Комарова понятно - там неверно отработали вещи которые уже были.
Аноним 18/04/22 Пнд 09:07:45 #445 №711421 
>>711419
Челленджера - нет, нельзя.

Колумбии в принципе можно, но это потребовало бы от НАСА полной мобилизации всех ресурсов в очень короткий срок.
Есть же отчет официальной комиссии, где подробно разобрана ситуация как если бы приняли решение, что повреждение не позволяет вернуться на Землю и готовили спасательную операцию.
Аноним 18/04/22 Пнд 09:21:15 #446 №711424 
>>711355
Из-за вакуума большой плотности энергии.
>>711382
На релятивистских, там же энергии бешеные.
Аноним 18/04/22 Пнд 09:26:29 #447 №711425 
>>711421
Я про то - можно ли было внести изменения в дизайны шаттла чтобы экипажи могли спастись?
Погибшие на Союзах погибли не из-за отсутствия систем а из-за неверного срабатывания - не раскрылся парашют и клапан открылся в стратосфере.
А у экипажей шаттла вообще не было каких-то возможностей спастись.
Аноним 18/04/22 Пнд 09:29:04 #448 №711427 
>>711425
Нет, нельзя.
Надо было полностью переделывать шатол.
Об этом тоже написано в официальных заключениях комиссий по Челенджеру и Колумбии.
Аноним 18/04/22 Пнд 10:32:47 #449 №711433 
>>711427
Понял-принял-услышал.
Жалко ребят, конечно.
А у Бурана вроде была система спасения двух пилотов, да?
Аноним 18/04/22 Пнд 11:32:43 #450 №711441 
>>711424
> Из-за вакуума большой плотности энергии.
Тоесть это циклический процесс?
Аноним 18/04/22 Пнд 11:48:28 #451 №711445 
>>711441
Нет. В самом начале эволюции Вселенной ее вакуум обладал очень большой плотностью энергии из-за квантовых эффектов. За счет этого вакуума началась инфляция - экспоненциальное расширение Вселенной. Инфляция продлилась 10^-35 сек, затем вакуум распался - его энергия перешла в рождение частиц, Вселенная заполнилась плотным и горячим веществом, с экспоненциального расширения перешла на степенное - этот момент называют Большим взрывом. С тех пор плотность энергии вакуума минимальна, однако ее хватает на то, чтобы обеспечивать ускоренное расширение Вселенной в наши дни.
Аноним 18/04/22 Пнд 13:14:32 #452 №711461 
image.png
>>711445
>затем вакуум распался
Не распался, иначе бы НИЧЕГО не существовало.
Аноним 18/04/22 Пнд 14:15:15 #453 №711472 
>>711461
Распался, но это был не тот распад, о котором ты думаешь. Он никак не связан с полем Хиггса. Там были осцилляции вакуума, т.к. вакуум большой плотности энергии - неустойчивое состояние, которое быстро рушится (так с любой физической системой, любая физическая система из состояния с большой плотностью энергии стремится перейти в состояние с минимальной плотностью). Этот распад привел к тому, что по итогу существуем мы, поскольку именно благодаря этим осцилляциям плотного вакуума во Вселенной появились частицы, т.е. вещество.
Аноним 18/04/22 Пнд 14:42:03 #454 №711473 
>>711433
>Катапультные кресла К-36РБ для "Бурана" были испытаны в условиях реального полета. Во время запусков беспилотных транспортных грузовых кораблей серии "Прогресс" с заводскими номерами 146-150 (при старте они получили названия "Прогресс-38...42") на головной обтекатель (ГО) ракеты-носителя (РН) вместо пустой оболочки системы аварийного спасения (САС) устанавливались катапультируемые кресла "Бурана".
>В пяти пусках "Прогрессов":
>- 10.09.1988 г. - "Прогресс-38";
>- 25.12.1988 г. - "Прогресс-39";
>- 10.02.1989 г. - "Прогресс-40";
>- 16.03.1989 г. - "Прогресс-41";
>- 05.05.1990 г. - "Прогресс-42"
>были проведены испытания с отстрелом катапультируемых кресел на активном участке выведения РН при заданных высотах (до 40 км) и перегрузках до 3,5 Маха. В результате экспериментов была подтверждена возможность применения открытого катапультируемого кресла К-36РБ.11Ф35 для спасения экипажа "Бурана".

>Слева на фотографии, сделанной членом поисковой группы Игорем Куликовым, катапультируемое кресло К-36РБ с манекеном космонавта в "бурановском" скафандре "Стриж" на месте падения после запуска "Прогресса-40".

https://www.buran.ru/htm/katapu.htm
Аноним 18/04/22 Пнд 15:12:33 #455 №711479 
Не знаю где спросить даже. Почти все гермошлемы, которые видел, имеют белый окрас, один раз видел фото якобы американских астронавтов в оранжевом шлеме, а сейчас даже его найти не могу. Можете пруфануть шлемы для скафандров/костюмов отличных от белого цвета прозрачные не в счёт?
Аноним 18/04/22 Пнд 22:58:36 #456 №711509 
IMG20220419005436162.jpg
Солнечный поглотитель зарегистрирован НАСА впервые более чем за 10 лет.
Объект размером с Юпитер перекачивал энергию Солнца, а затем улетал с невероятной скоростью.
Он вернулся 13.04. Внизу слева, в том же положении, что и в прошлый раз.

Почему нигде про это нет?
Аноним 18/04/22 Пнд 23:14:22 #457 №711512 
1601456965365.png
>>711509
> Почему нигде про это нет?
Потому что это очередной инфо-мусор. Какие-то процессы на солнце, которыми должны заниматься астрофизики, а не журнашлюхи.
Аноним 19/04/22 Втр 00:02:07 #458 №711520 
>>711384
Потому что для самолётных двигатель нужен окислитель, например.
Аноним 19/04/22 Втр 02:57:51 #459 №711533 
>>711384
>Почему в космос летаем на ракетах? Почему нельзя на самолёте?
Потому что на скоростях выше 5М топливо не успевает смешаться с воздухом и сгорать, а если воздух тормозить до дозвуковой - он дохуя нагревается и херит весь кпд.
Аноним 20/04/22 Срд 12:29:39 #460 №711662 
Есть какая нибудь книга с ответами на глупые вопросы для чайников, про чд, пространство, большой бадабум?
Читал как-то книжку Кипа Торна про интерстеллар, думал нормально объяснит но он начал писать про балки в десятимерных измерениях, а я даже не знаю что такое балки и из чего они вообще состоят, короче книга может быть и нормальная но не объясняет обычным людям ничего.
Аноним 20/04/22 Срд 13:09:11 #461 №711665 
>>711662
Да, всякие Каку и Тайсоны пишут именно такое.
Аноним 20/04/22 Срд 13:47:13 #462 №711669 
1280px-Nauticalsignalflags-USSBonhommeRichard(LHD6).jpg
Допустим у меня на космическом корабле за бортом вывешен флаг и я начинаю двигаться с ускорением - будет ли флаг развеваться в вакууме в обратную сторону моему движению? Допустим я перестал ускоряться и с включенными двигателями держу постоянную скорость - будет ли флаг развеваться в вакууме в обратную сторону моему движению?
Аноним 20/04/22 Срд 13:48:03 #463 №711670 
>>711662
https://www.labirint.ru/books/672091/
Аноним 20/04/22 Срд 13:49:05 #464 №711671 
>>711166
Эта картинка лживая, так как на Земле они сравнивают поверхностную воду, но в то же время на других планетах они сравнивают воду заключенную в недрах, которую даже только гипотетически предполагают. У Земли в недрах тоже дохуя воды заключено, так что не посчитать
Аноним 20/04/22 Срд 13:55:07 #465 №711674 
>>711669
Все завесит от характера и силы ускорения, ну и еще от механических свойств флага(упругость, вязкость, плотность).
Ну таки будет развеваться некоторое время после начала или конца ускорения.
Тепловые неравномерности флага будет сильнее шатать флаг, чем пуки корабля.
Аноним 20/04/22 Срд 13:59:31 #466 №711675 
>>711670
> Тут многие пишут, что книга написана предельно простым языком и будет понятна абсолютно каждому, далекому от физики… и я могу предположить, что эти отзывы написаны физиками)))
Аноним 20/04/22 Срд 14:16:45 #467 №711676 
>>711675
Я ее сам читал, она и правда написана простым языком, понятным неспециалисту, который не обладает знаниями по теме даже на уровне любителя. У Хокинга в принципе все книжки такие, он писал на широкую аудиторию. Так что можешь смело читать.
Еще Тайсон пишет в таком же стиле, но я его не читал, да и он мне не нравится, какой-то гумозный ниггер. Так что его рекомендовать не буду.
Аноним 20/04/22 Срд 16:13:02 #468 №711692 
Твойанус - это чёрная дыра или гас гигант?
Аноним 20/04/22 Срд 23:01:31 #469 №711727 
Когда перекатят спейс енгин тред?
Аноним 20/04/22 Срд 23:16:23 #470 №711728 
Почему Черенковское излучение только синее?
Аноним 21/04/22 Чтв 00:16:05 #471 №711732 
>>711676
Почитал, ну такое, ни о чём, про бога, про веру и очень мало про вселенную, вода вода вода, опять про бога и что его нет
Аноним 21/04/22 Чтв 06:41:00 #472 №711743 
>>711728
Тут завесит от среды, чем больше коэффициент преломления и меньше дисперсия, тем ближе к белому/желтому будет.
Интенсивность растет с частотой, а фотоны у нас порционные. В итоге длинноволновых фотонов значительно меньше, чем коротковолновых.
Аноним 21/04/22 Чтв 08:11:36 #473 №711744 
>>711732
Ну ладно, так уж и быть, тогда порекомендую книжку Тайсона, т.к. она предельно простая и только по теме астрофизики, без отвлеченных рассуждений на филасафские темы. https://vsenauka.ru/knigi/vsenauchnyie-knigi/book-details.html?id=53
Аноним 21/04/22 Чтв 08:15:29 #474 №711746 
>>711732
https://vsenauka.ru/knigi/vsenauchnyie-knigi/book-details.html?id=1171
Вот еще более предметная книжка Хокинга.
Аноним 21/04/22 Чтв 08:46:37 #475 №711747 
14699721380550.png
>>711692
Черная дыра. Сверхмассивная.
Аноним 21/04/22 Чтв 10:43:51 #476 №711754 
Испаряются ли ЧД планковской массы?
Аноним 21/04/22 Чтв 10:49:14 #477 №711755 
почему еще никто из астронавтов не слетал на Меркурий?
Аноним 21/04/22 Чтв 12:55:10 #478 №711762 
>>711755
Там слишком неуютно.
Аноним 21/04/22 Чтв 13:52:38 #479 №711766 
>>711762
Ночью можно лететь.
А в полярных районах есть области где Солнце никогда не жарит.
Аноним 21/04/22 Чтв 14:31:16 #480 №711768 
>>711766
На Луну 50 лет как слетать повторно не можем, а ты на Меркурий замахнулся.
>Солнце никогда не жарит
На Луну кстати высаживались по утрам (они там длинные), пока солнце не успело нагреть поверхность как сковородку.
Аноним 21/04/22 Чтв 15:11:36 #481 №711770 
>>711768
Реголит имеет очень низкую теплопроводность и теплоемкость, поэтому температура в 150(и даже 500) градусов не такая уж проблема, ибо ее обладает очень тонкий слой пыли. Главный источник тепла тут Солнце.
Утром высаживались по другой причине - чтоб не застать ночь, где без внушительного источника тепла померзнешь до -150 за неделю.
Аноним 21/04/22 Чтв 15:53:39 #482 №711774 
>>711744
>>711746
Спс
Аноним 21/04/22 Чтв 17:03:06 #483 №711776 
>>711509
>плазма, газы
>иррря оно круглое ды мама паглиди это нло
Аноним 22/04/22 Птн 02:42:01 #484 №711796 
Пацаны, какие ресурсы можно добывать из космоса?

Просто эпоха великих географических открытий состоит из колониальных войн, работорговли и вывоза нищтяков с земель папуасов (картофель, кофе, ганджубас и т.д.) - собственно почему все и начали вдруг интересоваться географией, строить и развивать суда, вкладываться в экспедиции. ПОКОРЯТЬ МОРЯ короче.

А какие профиты можно извлечь с солнечной системы? Типа лунный камень дробить и кирпичи из него делать? Или газ сосать с юпитера?
Аноним 22/04/22 Птн 10:47:25 #485 №711823 
>>711796
Иридии всякие из астероидов.
Аноним 22/04/22 Птн 10:57:23 #486 №711827 
>>711796
Капитилистическая логика тут не работает.
Энергетические затраты на транспортировку делают невозможным рост капитала при освоении солнечной системы.

Тут скорее средневековая экономическая система будет работать, когда инновации делаются только за счет накоплений когда они есть фактически. Ну и надо будет стремиться к максимальной автономии внеземных колоний. Как это будет экономически соотносится с тем, что они будут все равно критически зависимы от Земли, пока непонятно.
Аноним 22/04/22 Птн 11:23:21 #487 №711830 
>>711796
Суть добычи ресурсов в космосе в том, чтобы прямо в космосе строить всякие йобы, которые дадут другие гига йобы, а не возить сраные камушки на земляшку.
Но главный ресурс в космосе это внезапно энергия всякие йобы начиная с масштабов в квадратных километров полностью окупают себя.
Аноним 22/04/22 Птн 12:07:08 #488 №711836 
А какое электричество на МКС? Там же прямой ток, или инвертеры стоят для бытовых вещей со сраной земляшки?
Я думаю что это бред, скорее всего прямой ток и они для ноутбуков не везут эти кирпичи переводящие ток с переменного, верно?
Аноним 22/04/22 Птн 14:27:11 #489 №711853 
>>707999 (OP)
Возможно ли сделать вторую луну из земного мусора?
Аноним 22/04/22 Птн 14:40:57 #490 №711854 
>>711853
Возможно, но слишком накладно.
Аноним 22/04/22 Птн 14:41:22 #491 №711855 
>>711853
в принципе можна
Аноним 22/04/22 Птн 16:17:33 #492 №711856 
10-километровая комета. Последствия столкновения с землей?

Может есть визуализация в каком-нибудь планетарном движке?
Аноним 22/04/22 Птн 17:25:11 #493 №711864 
>>711856
> 10-километровая комета. Последствия столкновения с землей?
Зависит от плотности. В основном они охуенно рыхлые, с плотностью ~0.6г/см3, и состоят даже не из льда, а из замёрзших газов типа аммиака. Въебавшись в атмосферу такое ядро будет крошиться и испаряться столь быстро, что до поверхности разве что маленькие вмёрзшие камушки долетят. Будет много бумов типа Челябинского на огромной площади, с обширной бомбардировкой болидами. Но один не большой удар типа погубившего динозавров.

Однако комета комете рознь, и в ядре некоторых под толщей замёрзших газов может находиться вполне себе плотный 5-км силикатный астероид, а может и железо-никелевый. Вот это будет пиздец как в фильмах-катастрофах.
Аноним 22/04/22 Птн 17:39:36 #494 №711866 
>>711827
Ну можно ведь за счет всяких ёб науку толкать или открывать новые сферы. Сколько бы добыча не стоила, если добываемый ресурс ценен и не существует на земле - он будет окупаться.

В конце концов ротшильд захочет себе дворец из лунных кирпечей - че ты ему сделаешь? Есть заказ - есть прибль, что еще нужно всем этим инженеграм чтоб занятся такой хуйней?
Аноним 22/04/22 Птн 21:20:27 #495 №711883 
>>707999 (OP)
Я только что видел комету, минут 5 назад? Локация Йошкар-Ола, на юго-западе, 30-50 градусов над горизонтом. Маленькая тоненькая линия быстро проскочила, не очень светлая.
Аноним 22/04/22 Птн 21:21:24 #496 №711884 
>>711856
на юкатан вроде как раз 10км упала и убила диносов
Аноним 23/04/22 Суб 04:41:19 #497 №711921 
>>711883
Точно не комету. Болид, может быть.
Или спутник но наврядли.
Аноним 23/04/22 Суб 08:40:19 #498 №711936 
>>711883
> Маленькая тоненькая
Прямо как твой хуишко
Аноним 23/04/22 Суб 13:10:37 #499 №711958 
2786475c9b4fbbdfb5575f2f5a13cd4a.jpg
>>707999 (OP)
Сосач, объясни мне одну вещь: как инженеры планируют заставить аэростаты летать в атмосфере газовых гигантов? Она же сама по себе состоит из водорода и гелия
Аноним 23/04/22 Суб 13:22:56 #500 №711960 
>>711958
Нагревать, епта.
А вообще проще орбитальными зеркалами создать горячее пятно, которое будет фонтанировать плазмой, которая собирается орбитальным магнитом.
Аноним 23/04/22 Суб 15:17:14 #501 №711964 
>>711960
Газ же плохо нагревается от света
Аноним 23/04/22 Суб 15:57:06 #502 №711967 
>>711958
Я бы зассал там быть. Чуть что не так - ты падаешь в бесконечную бездну и ничего не поделать, ты обречен на гарантированную смерть и от тебя ничего не останется.
Аноним 23/04/22 Суб 16:25:43 #503 №711969 
>>711967
Да я и сам бы даже на гравитационный маневр к Юпитеру не приблизился
Аноним 23/04/22 Суб 17:20:10 #504 №711978 
>>711964
Это как светить.
Накачка газовых лазеров это обычные дуговые источники света.
В случае вешних слоях атмосферы при низких давлениях становится возможна ионизация через последовательные переходы. Даже частично ионизованный газ считай поглощает свет почти во всех диапазонах.
В случае Юпитера еще играет фактор сильного магнитного поля, который позволяет разогретый ионизированный перейти в когерентный режим - можно будет сделать мини-джет.
Аноним 24/04/22 Вск 13:37:24 #505 №712034 
Училка сказала, что как только каробль землян плюхнулся на луну, начали отлетать камушки и с тех пор отлетают и скоро луне и всем нам пиздец, я боюсь, это првда?
Аноним 24/04/22 Вск 13:56:28 #506 №712036 
>>708482
Да че ты ему объясняешь. Видно же типикал невылезатор
Он из дома даже выйти боится, потому что его там в жопу выебут
Аноним 24/04/22 Вск 14:48:15 #507 №712044 
1650800891586.png
>>712034
Абсолютная
Аноним 24/04/22 Вск 16:58:51 #508 №712052 
>>712036
>Он из дома даже выйти боится, потому что его там в жопу выебут
Но ведь я дома как раз и сижу, потому что круглыми сутками ебу в жопу твою мамашу, а она скулит как последняя сучка.
Аноним 24/04/22 Вск 17:38:22 #509 №712053 
Что произойдет с человеком вне магнитосферы земли?

Даже не на марсе, а просто пролетев чуть дальше луны, людишкам комфортно будет?

Как наса отвечает на вопросы об радиации на марсе, готовясь туда титушек отправлять?

Каких размеров должен быть кораблик, чтоб потом обратно на землю вернутся? Или планируется просто отправлять туда людей как на маяк посреди океана - на пожизненную вахту. Типа вы теперь КОСМО МОРЯКИ, на марсе всю свою оставшуюся жизнь проведете - а при смерти будут просто замену вместе с припасами присылать?
Аноним 24/04/22 Вск 17:47:07 #510 №712054 
>>712052
Как же ты полыхаешь уже месяц, божечки, как же забавно.
Аноним 24/04/22 Вск 19:24:29 #511 №712062 
>>712054
То есть на ебало ссут тебе, а "полыхаю" я? Ладно.
Аноним 24/04/22 Вск 20:41:39 #512 №712068 
мнекажется или звезды ниже к горизонут меркают интенсивнее? у меня плохое зрение
атмосфера/турбулнетонсти? переломление света больше т.к. толще атмосефра под углом?


Алсо, почему, когда солнце только зашло, нА ВОСТОКЕ небо все еще ярко-голубое? Откужа так много света там, на востоке?
Аноним 24/04/22 Вск 21:03:59 #513 №712071 
блять я только что видел
одна типа звезда(саомлет) двигаалась по нему, пока шел за боноколем - 10-15 сек - начала двигаться в дургую сторону


не истребетль
не вертолет

что-то неземное

Аноним 24/04/22 Вск 21:11:53 #514 №712072 
>>712071
Общедоступный интернет был ошибкой.
Аноним 24/04/22 Вск 21:18:43 #515 №712073 
>>712053
А еще, были ли скандалы с сексом на мкс? Там же и бабы есть и условия крайне романтические. Да и алкашку туда протаскивают периодически.
Аноним 24/04/22 Вск 21:28:43 #516 №712074 
>>712073
Скандалов не было.
Аноним 24/04/22 Вск 21:46:14 #517 №712076 
>>712053
Мне кажется эволюцияну или бох просто не задумывала нас как покорителей космоса, мы были созданы и заточены под выживание на отдельно взятой планете - и даже пиля самый технологичный и современный аппарат для дальних полетов в нем всегда будет балласт и изъян, самая слабая часть механизма - гомо сапиенс.
Даже зачатков, таких слабых росточков, которые можно было бы нарастить хотя бы искусственно, например устойчивость к той же радиации - у нас просто нет. Человек заточен ебать все живое в пределах земли, но дальше неё - делать ему нехуй.
Аноним 24/04/22 Вск 23:57:11 #518 №712083 
>>712073
> А еще, были ли скандалы с сексом на мкс? Там же и бабы есть и условия крайне романтические. Да и алкашку туда протаскивают периодически.
Ты представь как там воняет. Бабы и так славятся своим чуханством, они все ТНУС и наверняка подходят подготовку как правильно срать, прежде чем лететь на мкс по квоте. Там не до ебли будет, точнее, только извращенец захочет.
Аноним 25/04/22 Пнд 02:19:07 #519 №712088 
>>710736
Опять маняпроекции
Аноним 25/04/22 Пнд 08:02:31 #520 №712093 
>>712062
Тебя весь спейсач уже обхаркал, как ты и любишь, можешь уёбывать.
Аноним 25/04/22 Пнд 11:44:25 #521 №712101 
>>712093
Ну то есть опять-таки на ебальник ссут тебе, а "обхаркал весь спейсач" меня? У тебя явные беды с логикой, хотя и не только с ней.
Аноним 25/04/22 Пнд 13:40:56 #522 №712108 
>>712101
Шиз, таблетки.
Аноним 25/04/22 Пнд 15:08:42 #523 №712118 
>>712108
За щеку отсыпал тебе таблеток, проверяй.
Аноним 25/04/22 Пнд 21:11:36 #524 №712132 
image.png
Было бы ли удобнее в космосе выращивать кристаллы типа сапфира?
Аноним 25/04/22 Пнд 23:22:56 #525 №712144 
>>712132
Да. Но в космосе становится выгодно плазменное осаждение вместо вытягивание кристалла из расплава.
Аноним 26/04/22 Втр 11:21:20 #526 №712173 
>>712144
А при плазменном осаждении получится ли буль типа той картинки или как оно вообще? Планы были?
Аноним 26/04/22 Втр 11:40:45 #527 №712174 
>>712173
Вполне, даже больше. Через плазменное осаждение можно получать огромный монокристалл высокой чистоты. Только вот в земных условиях это нахуй надо, слишком уж дорогой выйдет хуйня. Поэтому плазменное осаждение применяют для производство тонких пленок, полупроводников и прочей экзотической хуйни вроде графена.
Планов много, только вот еще на земле эту технологию еще не откатали.
Аноним 26/04/22 Втр 12:14:44 #528 №712178 
>>712174
Я что-то читал про искусственные алмазы высокой чистоты получаемые осаждением. Палятся по идеальной чистоте, в отличие от естественных где примеси даже в самых охуенных есть.
Аноним 26/04/22 Втр 23:11:39 #529 №712205 
Что там с буксиром?
Аноним 27/04/22 Срд 10:42:39 #530 №712246 
Почему все еще не оборудовали всякие МКСы искусственной гравитацией?
Надо де ПРОСТО "крутилку" присоединить к ракете?
Слишком тяжело для взлета? А если частями на орбиту отправлять?
Аноним 27/04/22 Срд 11:50:32 #531 №712251 
>>712205
Он заземлен.
>>712246
МКС достаточно старая, чтобы на ее крутилку ставить. Нужно дизайнить и строить с нуля новую станцию.
Аноним 27/04/22 Срд 11:55:09 #532 №712253 
>>712251
>МКС достаточно старая, чтобы на ее крутилку ставить.

Скрипеть будет?
Аноним 27/04/22 Срд 13:46:57 #533 №712258 
>>712253
В некотором роде да.
Нынешняя мкс довольно неповоротливая громоздкая хуита, для которой требуется целая система коррекции, чтобы она не вошла в разрушительный режим автоколебаний при маневрах ориентации.
Установка крутилки усложнит все на порядки, да еще потребует оптимизацию момента инерции.

Ну еще МКС достаточно большая, что бы "чувствовать" прилив от Земли. Если еще раскрутить ее, то прилив станет сильным источником вибраций.
Аноним 27/04/22 Срд 18:44:31 #534 №712267 
>>712068
БАМП
Аноним 28/04/22 Чтв 01:00:17 #535 №712284 
>>712283 (OP)
>>712283 (OP)
>>712283 (OP)
Перекот
comments powered by Disqus

Отзывы и предложения